You are on page 1of 201
Past Papers MRCP One January 2021 pu les alo (0 Gguust y Dr. Murtadha Hussein Abbas a Bre Jay Yael Question 1 of 100 A 64-year-old man presents to the Cardiology Clinic for review. Despite treatment with bisoprotol 10 mg, ramipril 10 mg, furosemide 40 mg and eplerenone 25 mg, he still complains of shortness of breath. He is only able to climb the stairs once per day and finds it difficult to walk 100 m to the bus stop. His blood pressure is 110/75 mmHg and pulse is 74 bpm and regular. He has crackles at both bases on auscultation of the lungs. There is no ankle swelling Investigation Result Normal value Haemoglobin (Hb) 120 g/t 135-175 gil White cell count (WCC) 67x 10% 4-11 x 10% Platelets (PLT) 274x109 150-400 x 10% Sodium (Na+) 137 mmol/L 135-145 mmol/l Potassium (K+) 4.3 mmol 3.5-5.0 mmo Creatinine (Cr) 133 pmol/l 50-120 pmol/l Echocardiogram Ejection fraction (EF) 30% Which of the following is the most useful next step in the patient's management? A Increase eplerenone B Increase furosemide c Start ivabradine D__ Start low-dose morphine E Switch ramipril to sacubitrivalsartan tion Save and Leave End Session Skip Question Blog About Pastest ContactUs Help © Pastest 2023 Question 1 of 100 Co He is only able to climb the stats once per day and finds itficut to walk 100 m tothe bus stop. His blood pressure 110/75 mmHg and pulse is 74 bpm and regular. He has IGRGESEEREN BSEES on auscultation of the lungs. There is FORAKEISWEUIAG. 4 Investigation Result Normal value : ‘White cell count (WCC) 67 «10% 411% 10% Platolets (PLT) 274 x10: 150-400 10°21, as bb bb} Sodium (Nas) 137 mmott 135-145 mmol Potassium (K+) 43 mmol 35-50 mmolt ia tesa Creatinine (C4 133 pout 50-120 pmol & 5 Echocardiogram Ejection fraction (EF) 309% oz 5 Which of the folowing i the ' ca 5 ich ofthe following isthe most useful next step inthe patient's management? Qs C 95 0 ‘A Increase eplerenone oS F B Increase furosemise q Oo 98 D CStartivabradine ° D Start low-dose morphine ° «| eee cee qa Explanation cd Senet Compared to angiotensin-converting enzyme inhibition inpatients with end-stage heart failure, sacubitrlValsartan combination therapy reduces heart failure admissions or death by 208 itis thought that this improvementin outcomes occurs because sacubitil reduces degradation ofa range of vasoactive peptides which may impact positively on outcomes A. Increase eplerenone Increasing the eplerenone dose further here is kely to have a limited effect on the symptoms of heart failure ‘and could increase the risk of hyperkalaemia, Studies have shown that most ofthe effectiveness of mmineralocorticoid receptor antagonists in heart failure is gained at low dose B Increase furosemide Although furasemide i effective in reducing symptom of uid overload inthe short term, it does not have an effect on long-term outcomes in heat failure. There are also no features here that imply significant fluid ‘overload where an increase in diuretic therapy is likely to be needed C Startivabradine Ivabradine is only of value in treating patients who have heart failure with a low EF and a resting heart rate above 75 bpm. n this case, with a heart rate of 74 bpm on maximal beta-blockade,ivabradine is not indicated, It can be used with or without beta-blockade where it reduces mortality and hospitalistions due toeart fallure by 189, D Start low-dose morphine Starting low-dose morphine reduces preload ands of value inthe treatment of heart failure symptoms in end-stage disease. tis, however, potentially best deployed once other options have been exhausted. In this case, the option of switching ramipril for sacubitrWalsartan remains the most useful ag Question Feedback nd Review Blog About Pastest Contact Us Help 4 © Pastest 2023 Question 2 of 100 ‘73-year-old man is reviewed on the Medical Admissions Ward after suffering a complex partial seizure. A decision is made to start him on carbamazepine. He is also taking long-term warfarin therapy, having been diagnosed with atrial fibrillation four years ago. Which of the following is the most likely to happen to this patient's international normalised ratio (INR)? A Fallin his INR maximal over 3-4 weeks B Fall in his INR maximal over 2-3 days No change in his INR D Rise in his INR maximal over 3-4 weeks E Rise in his INR maximal over 2-3 days Previous Question Save and Leave End Session Skip Question Blog About Pastest ContactUs Help © Pastest 2023 Question 2 of 100 Cog {A73-year-old mans reviewed on the Medial Admissions Ward after suffering a complex partial seizure. A a decison is made to start him on ESRBSMREBHIE. He is also taking Long-term WSRATNRBRSBY.naving been diagnosed with ara rilation four years ago. Poor Responses % Which ofthe following i the most ikely to happen to this patient's international normalised ratio (INR)? te ent FE B Fallin is INR maximal over 2-3 days No change in is NR ee a D__isein his INR maximal over 3-4 weeks g a @ oO E __Riseinhis INR maximal over 2-3 days a Oo Explanation * co Oo H - = ne nae 6 oO Carbamazepine is a potent cytochrome P450 enzyme inducer across a range af P40 enzyme subfamilies, including 142, 286, 2C9, 2C19 and 344, It also exhibits autoinduction, which builds over a number of a oO ‘weeks, This leads to a need to Increase the dose for treating epilepsy after around three weeks. What this ‘means in terms of INR is thatthe INR progressively falls because of a reduction in warfarin efficacy over the 98 Oo course of 3-4 weeks 2 Fallin his INR maximal over 2-3 days —_ ° Although an inital fall inthe INR is seen over 2-3 days, because P4S0 induction for carbamazepine builds lap ac senees slowly, the trough inthe INRis seen at the 3- to 4-week time point. is important therefore to monitor the a INR regularly and titrate the dose accordingly in patients who start therapy with carbamazepine © Nochangein his IR © 6 Carbamazepine is a potent P450 enzyme inducer across multiple PASO enzyme subfamilies, For this reason, “oar itis very important to monitor the effects or levels of drugs that ae highly dependent upon P45O ‘metabolism. Good examples include ciclosporia, the levels of which can fall rapidly on beginning carbamazepine and warfarin, where the INR. can become rapidly subtherapeutic when carbamazepine is commenced, Rise inhi INR maximal over 3-4 weeks ‘Arise in the INR is seen in patients prescribed P450 enzyme inhibitors. Warfarin is metabolised via the 1A2 and 23 (s-warfarin} P40 subfamilies. Examples of potent 2C9 inhibitors include agents such as fluconazole and cimetidine and 12 inhibitors include cimetidine and ciprofloxacin. Changes in bowel flora induced by antibiotics may also lead to an increase in the INR. E Rise inhis NR maximal over 2-3 days Potent cytochrome P50 inhibitors ae associated with a rapid rise inthe INR over 3 few days because of reduced metabolism of warfarin. This is particulary true when warfarin is co-prescribed with inhibitors of 1A2 and 209, fatiequeton: @ Previous Question ag Question Feedback nd Review Blog About Pastest ContactUs Help a © Pastest 2023 a Question 3 of 100 A 24-year-old woman is admitted to the Emergency Department following a collapse at home with nausea and vomiting. She has been ill for the past three days with a heavy cold. She has type | diabetes, for which she takes a basal-bolus insulin regimen. Her blood pressure is 105/70 mmHg and her pulse is 105 bpm and regular. Her respiratory rate is 35 breaths/minute. Her abdomen is generally tender, with no masses, and she looks thin. You can see injecting sites for her insulin which have been properly rotated across her abdomen. Investigation Result Normal value Haemoglobin (Hb) 102 g/t 115-160 g/t White cell count (WCC) 13.7 x 10/1 4-11 x 10% Platelets (PLT) 301 x 10% 150-400 x 10%/L Sodium (Na*) 140 mmol/L 135-145 mmol/L Potassium (K*) 5.9 mmol 3.5-5.0 mmott Creatinine (Cr) 144 molt 50-120 umol/t Bicarbonate (HCO;") 15 mmoUt 23-30 mmol/t Glucose 43.2 mmol/L < 11.1 mmolft Urine dipstick Positive for glucose and ketones Which of the following is the most important initial step in this patient's management? A litre of 0.9% saline over one hour B_ 8 units of subcutaneous (SC) rapid-acting insulin C Intravenous (IV) calcium gluconate D Salbutamol nebuliser E 10 units of Actrapid® in 50 ml of 50% dextrose Previous Question Save and Leave End Session Skip Question Blog About Pastest ContactUs Help © Pastest 2023 Question 3 of 100 {A 24-year-old woman s admitted to the Emergency Department following = SESS RRS WR SUES ‘RRAVoRnitng. She ha boon HURSFRS)SSSERSeWSYSWRERIYEBTE Sho has type | ciabetes for which she takes basal-bolus insulin regimen. Her bload pressure is 105/70 mmHg and her pulse is 105 bpm and eer Responees % regular. Her respiratory rat is 35 breathsinin.te, HESDSORERISTGGRSUV AGRE with no masses, and she looks thin You can see injecting sites for her insulin which have been properly rotated across her abdomen | Investigation Resutt Normal value . Haemoglobin) 102 A 115-1609 : White cellcount WC) 137 «20% sano . Platelets (PLT) 301 x 10% 150-400 x 10% - . aa ‘Sodium (Na*) 140 mmo 135-145 mmol/l g oe a 0 Potassium (°) 59 mmolA 35-50 mmott @ 0 Crestinine (C9 144 uot 50-120 ymotA a Oo Bicarbonate (HCO>>) 15 mmol 23-30 mmott a 0 Glucose 432 mmolh < 11.1 mmolt a Usine dipstick Positive for glucose and ketones & oO : a 0 Which ofthe following isthe most important inal step in this patents management? a 0 ° 8 Gunite of subcutaneous (SC) rapid-acting insulin Dae reteeinse C Intravenous (V) calcium gluconate D_Salbutamot nebuliser E 10 units of Actrapid® in 50 ml of 50% dextrose Explanation * ‘Tris patient has diabetic ketoacidosis (OKA), potentially asa result of her recent respiratory tract infection. fen at the time of presentation, patients have relative dehydration of 5 litres or more, The most important intial intervention is fluid replacement and normal saline is alsa appropriate intially, Although Kis high, serum K* usually falls rapidly once the fluid deficit is corrected and insulin is commenced. Typical deficits in DKA in adults are water 100 ml/kg, Na" 7-10 mmal/k, Cl- 3-5 mmoUkg and K* 3-5 mmoUkg, B Buns of subcutaneous (SC) rapid-actng insulin Although management of the patient's insulin needs isa crucial part ofthe treatment of DKA. a fixed-rate IV insulin infusion calculated on the basis of 0.1 unts/kg body weight per hour is recommended. This is expected to drive a reduction in glucose and a fallin ketones of approximately 05 mmol per hour. In the event ketones do not sufficiently improve, the rate can be increased. C Intravenous (1) calcium gluconate Intravenous (1) calcium gluconate is indicated forthe treatment of life-threatening hyperkalaemia. A level of 5:9 mmolA only represents a moderate elevation n this situation, with rapid fluid replacement and institution of o fixed-rate insulin regimen," is Uikey to fall very rapidly. D_Salbutamot nebuliser {A salbutamol nebuliser isnot likely to be required for this patient, Salbutamol is usualy considered as an acute intervention for life-threatening hyperkalaemia while lV access is obtained, so that other interver can be instituted, K* wil al rapidly once fluid and insulin replacements are instigated, E 10 units of Actrapid® in 50 ml of 50% dextrose {rapid glucose-insulin infusion, typical of that used in life-threatening hyperkalaemia is not required here. Rpid replacement of the fia deficit, coupled with the fixed-rate insulin infusion, should be enough to drive a fain serum K". Rapid instigation of lV normal saline is the mast important frst step here Previous Question ag Question Feedback nd Review Blog About Pastest Contact Us Help © Pastest 2023 a% OR bol triad Question 4 of 100 A 34-year-old woman presents to the Endocrine Clinic with galactorrhoea, vaginal dryness and loss of libido. She can express milk on minimal nipple stimulation and has not had a menstrual period for four months Investigations: Investigation Result Normal value Prolactin 5200 < 700 miu/ Magnetic resonance imaging Pituitary adenoma with pressure (MRI) brain scan on the optic chiasm Which of the following visual defects is most likely to be found on visual examination? A Binasal hemianopia B___Bitemporal hemianopia C Central scotoma D___ Left homonymous hemianopia E Loss of colour vision Previous Question Save and Leave End Session Skip Question Blog About Pastest ContactUs Help © Pastest 2023 Question 4 of 100 ‘A 34.year-old woman presents tothe Endocrine Clinic with galactorthaes, vaginal dryness and loss of Uibide. She can express milk on minimal nipple stimulation and has not had a menstrual period forfour months, Investigations: Investigation Result Normal value $700 miu Which ofthe folowing visual defects is most likly to be found on visual examination? A Binasat hemianopia } s [ornare | © Central scotoma Left homonymous hemianopia E Loss of colour vision Explanation * Je [eevcnmmmm Pressure on the optic chiasm leads to loss of temporal vision affecting both the left and right visual fields ‘Tris is because the image you see is reversed by the ens before it reaches the retin, A Binasat hemianopia Binasal hemianopia i a very rare visual fleld defect, described in association with a number of conditions, including bilateral internal carotid artery aneurysms, hydrocephalus, intracranial mass lesions and elevated intracranial pressure tis not associated with a pituitary tumour. © Central seotoma {A central scotoma i a blind spotin the centre ofthe visual field, Causes of a central scotoma include macular degeneration, demyal ‘as exnosure to methanol. Scatomas may be seen in women with pre-eclampsia, in association with severe hypertension id other D_ Left homonymous hemianopia Left hamenymous hemianapia occurs because of a lesion posterior tothe optic chiasm. tis seen because of @ lesion tothe right optic tract orto the right occipital labe tse. E Loss of calour vision Loss of colour vision 0 srs as 2 result of pressure on the optic nerve from conditions such as Graves ‘ophthalmopathy or from raised intracraniat pressure, eg in patients with idlopathc intracranial hypertension, Previous Question ag Question Feedback nd Review Blog About Pastest Contact Us Help © Pastest 2023 a a @ 3 Qs 95 ry qr 8 eee eesees Answered Flagged 0 [ 0 Oo 0 0 0 Mo —_ © qa see Sy Pe Jesl PY aso Question 5 of 100 ‘51-year-old woman with a long history of depression and alcoholism is admitted unconscious to the Emergency Department, having been found with an empty bottle of amitriptyline, an empty bottle of whisky and a suicide note, Her blood pressure is 95/70 mmHg and her pulse is 105 bpm and regular. While you are examining her, she suffers a short tonic-clonic seizure. An anaesthetist is called to assess her for intubation and ventilation. Investigation Result Normal value Haemoglobin (Hb) 121 git 115-160 g/l White cell count (WCC) 10.0 x 10% 4-11 x 10% Platelets (PLT) 267 x 10% 150-400 x 1027/1 Sodium (Na*) 130 mmol 135-145 mmol/l Potassium (K*) 5.1 mmol/l 3.5-5.0 mmolt Creatinine (Cr) 132 ymolft 50-120 umol/t Bicarbonate 15 mmol/t 18-33 mmol/L An electrocardiogram (ECG) was carried out. The results were: sinus rhythm, QRS 115 ms. Which of the following is the most appropriate next step? A 10 ml of 10% calcium gluconate B 100 mlof 8.4% sodium bicarbonate C 250 ml of 1.8% sodium chloride D 500 ml of 1.26% sodium bicarbonate E 1000 ml of 0.9% sodium chloride Previous Question Save and Leave End Session Skip Question Blog About Pastest ContactUs Help © Pastest 2023 Question 5 of 100 Cog ‘A1L-year-old woman with along history of depression and alcoholism is admi Emergency Department, having been found wth an PBR Bo aie pine EPR BOUL oF WHER) (BRLBSURWIHEEE Her b1000 pressure fs 95/70 mmHg ang her pulses 105 bpm and regular. While you are er Respones examining er, he suffers 2 SRRRERORISRIOAESRUG. An anacstneti is called to assess her for intubation and ventilation, 4 Investigation Resutt Normal value Es ‘Haemoglobin (Hb) 121g 115-160 9h “ White call count (WC) 100% 10% 411x107 Patetts (PLT) 267 x10 150-400 10% [ak bb ee Soci (Na) 130 mmout 135-145 mmolA Potassium {K*) 5.1 mmout 35-50 mmol a Answered Flagged Creatinine (Co) 132 pmollL 50-120 pmol a o carbonate 15 mmol 16-23 mmout ae OD ‘An electrocardiogram (ECG) was carried out. The results were: sinus rhythm, QRS 115 ms. e 0 Which of the folowing i the most appropriate next ste? a o : = @s o 96 Oo A omLof 10% calcium gluconate a Oo ener - ae Oo a ° © _250mlof 1.8% sodium chore 1D S00mLof 1.26% sodium bicarbonate ° E 1000 ml of 0.9% sodium chloride a Explanation o eB Ezz ; Even in the absence of marked acidosis, sodium bicarbonate is fective in reducing the risk of tachyarrhythmias and seizures in patients with a tricyclic antidepressant overdose. Ibis given as a bolus of 50-100 ml of 8.4% solution, witha targ conventional antiarthythmics and antepitepties at reducing the risk of arrhythmias and seizures, respectively pH of 7.45-7.55. Sodium bicarbonate is more effective than A 10 mlof 1096 calcium gluconate Calcium gluconate is used as an emergency measure with respect to carcioprotection, where patients have life-threatening hyperkalaemia and ECG changes consistent with a high risk of ventricular tachycardia ancoe cardiac arrest tis given as a bolus and may be repeated after 15-20 minutes. C 250.miof 1.8% sodium chloride Although Na* is low at 130 mmol. tis the tricyclic antidepressant overdose, rather than hyponatraemia, ‘hati likly to be responsible fo this patient's seizures, Sodium chloride 1.8% is only given for hyponatraemia when Na* isifethreateningly low and Na* can be carefully monitored in the Intensive Therapy Unit setting, D500 ml of 1.2696 sodium bicarbonate Although 500 ml of 1.2696 sodium bicarbonate may be effective in elevating pH, it will take some time to infuse and patients with tricycic antidepressant overdose may be sensitive to volume shifts. For this reason, 1 smaller volume of 8.4% solution is preferred E1000 mi of 0.996 sodium chloride Tris patients low Na* is not her most important problem. The most important next step is managing the QRS prolongation and seizures, For this reason, 8.4% sodium bicarbonate isthe preferred intervention Previous Question ag Question Feedback nd Review Blog About Pastest Contact Us Help 4 © Pastest 2023 OES Bon byl raed Question 6 of 100 A 32-year-old South-Asian woman presents to the clinic for review. She complains of chronic bone pain and weakness of her hips, which makes it difficult for her to climb the stairs. She follows a vegetarian diet and rarely leaves the house. Her blood pressure is 148/84 mmHg and her pulse is 84 bpm and regular. Her body mass index is 27 kgim?. She has a 4/5 weakness of her hip flexors. Investigations: Investigation Result Normal value Haemoglobin (Hb) 100 g/t 115-155 g/t White cell count (WCC) 6.1% 10% 4-11 x 10% Platelets (PLT) 209 x 10%/1 150-400 x 10%/L Sodium (Na*) 144 mmol/L 135-145 mmol/l Potassium (K*) 4.3 mmol 3.5-5.0 mmol Creatinine (Cr) 102 pmol 50-120 pmol/L Calcium 2.1 mmol 2.1-2.65 mmol Phosphate (PO«) 0.7 mmol 0.8-1.5 mmo Alkaline phosphatase 210 IU/L 30-130 IU/L Parathyroid hormone 130 ng/t 12-72 ngh Which of the following is the most likely cause of her proximal muscle weakness? A Hypothyroidism B Osteomalacia C Osteoporosis D _ Polymyalgia rheumatica E Polymyositis Previous Question Save and Leave End Session Skip Question Blog About Pastest ContactUs Help © Pastest 2023 suv a7 et dee ao Seo (ote aste) ere ‘A 32-year-old South-Asian woman presents to the clinic for review. She complains of which makes Ielfcul for ert climb the stars. She follows a Her blood pressure is 148/84 mmHg and her puse is 64 bom and regular, Her body eer Respones 9 mass index 27 kan’, She has a MiSWSRAGEEOFRSERISMER investigations Investigation Result Normal value White cel count (WCC) 61x10 411x210 : Platelets (PLT) 209 « 1027 150-400 10°” 0 0m» w mm 00 Sodium (Na") 144 mmol 135-145 mmolA Q Answered Flagged Potassium (K*) 43 mmott 35-50 mmol cr Oo Creatinine (Cn 102 pmott 50-120 pmol 2 Cateium 2.1 mmout 2.1-2.65 mmol o i Phosphate PO.) 07 mmol 08-1.5 mmolt a Ld ‘Alkane phosphetase z101Un =o a ( arethireid Korman 1300! IT nol 95 0 Which ofthe following is the most likely cause of her proximal muscle weakness? 6 Oo Q o os ( ° A Hypothyroidism Je fommne —e v a C Osteoporosis D__Polymyalala rheumatica G E Polymyosits Explanation ° Jeon ‘This patient eats a vegetarian diet which can be Law in vitamin D and lead to decreased calcium absorption from the gastrointestinal tract. Coupled with limited time spent outside, this has led to Low vitamin D, a calcium level atthe lower end ofthe normal range, low phosphate and raiced alkaline phosphatase, with 3 reactive increase in parathyroid hormone, Ths clinica piture is consistent with osteomalacia. Callum and vitamin D supplementation should lead to the resolution of the patient's symptoms ‘A Hypothyroidism ‘Although hypothyroidism can lead to proximal muscle weakness, the blood picture here with low calcium, phosphate, elevated alkaline phosphatase and parathyroid hormone fits with osteomalacta C Osteoporosis Osteoporosis is characterised by a reduction in bane mineral density in the presence of a normal blood workup, It does not, therefore, fit withthe clinical picture seen here, D_Polymyalaia rheumatica Polymyalaia rheumatica is associated with pain and stifress, particularly in the morning. Muscle weakness. is Less common, and its seen in much more elderly patients vs this relatively young woman. It is also nat associated withthe abnormalities of calcium metabolism seen here E _Polymyosits Polymyosits leads to proximal muscle weakness with increased creatine kinase. tt does not lead to the abnormalities of calcium metabolism seen here. Given the blood picture and the patient's clinical history, the scenario is consistent with osteomalacia, Previous Question ag Question Feedback nd Review Blog About Pastest Contact Us Help a © Pastest 2023 Question 7 of 100 An 18-year-old student presents to the Emergency Department passing gross haematuria. He tells you that he began suffering from a sore throat 2-3 days earlier and has a cough. He has no medical illnesses and takes no regular medication. His temperature is 37.9 °C and his blood pressure is 110/70 mmHg, He has evidence of pharyngitis. His chest is clear. His abdomen is soft and non-tender. Investigations: Investigation Result Normal value Haemoglobin (Hb) 142 git 135-175 g/l White cell count (WCC) 10.1 x 10% 4-11 x 10% Platelets (PLT) 391 x 10% 150-400 x 10%/t Sodium (Na*) 144 mmol/L 135-145 mmol/L Potassium (K*) 3.7 mmol 3.5-5.0 mmol/L Creatinine (Cr) 89 pmol/L 50-120 umol/t Urine Blood +++ Which of the following is the most appropriate next step? A Aspirin 75 mg B Azathioprine 50 mg Cc Observation D___Prednisolone 40 mg E — Ramipril2.5mg Previous Question Save and Leave End Session Skip Question Blog About Pastest ContactUs Help © Pastest 2023 Bsr aot Question 7 of 100 Co {An 18-year-old student presents tothe Emergency Department passing SRSSSERLIRE. He tells you that Is he began suffering rom aoa RASEES Ways eatise arg SETEBUBH. He has no meical illnesses and takes no regular medication. His temperatures 379°C and his blood pressure 110/70 mmHg. He has ARERR H's crest is clear. His abdomen is soft and non-tende. Investigations: Investigation fesut ee @ Anwwered Fagged Pras 37 molt 35-50 mmol Qi vO ro] Blood + ka i a Oo Which ofthe following ithe most appropiate ext step? a 5 SS 5 D A Aapin 750 96 5 q Oo B Azathioprine 50 ma : Ss ° Prednisolone 40 mg o Ip Neptcpaty a E —Ramipril 25mg Explanation o Be ‘This patient has haematuria in close proximity to an upper respiratory tract infection This is suggestive of immunoglobulin & (lg) nephropathy, His blood pressure is normal and there is ne proteinuria, This fits with a good prognosis and intervention with corticostercids is, therefore, not required at this stage (although both proteinuria and blood pressure should be monitored, A Aspirin 75 mg ‘The proximity ofthis patient's haematuria its with an underlying diagnosis of immunoglobulin A (IgA) nephropathy. Aspirin may only be of value inpatients with progressive renal disease B Azathioprine 50 mg ‘Azathioprine is not needed here, given this patient has IgA nephropathy with a good prognosis since there is ‘an absence of proteinuria, tcan be considered in combination with corticosteroids for symptoms that fal to resolve D_Prednisolone 40 mg Prednisolone is indicated for the treatment of IgA nephropathy when there is significant proteinuria, where corticosteroids have been proven to stow the progression ofthe disease. In this case, the absence of proteinuria is associated with a good prognosis. E Ramipril 25mg ‘Tis patients blood p underlying diagnosis, caries a good prognosis and there is ne indication for angiotensin-converting enzyme inhibition ure is normal and there is ne proteinuria, In this case, IgA nephropathy, the Blog About Pa 4 © Pastest 2023 Contact Us Help see Sy Bory byl raed Question 8 of 100 Anew testis under development for the detection of inflammatory bowel disease. The performance characteristics of the test are shown below: Inflammation seen on colonoscopy —_No inflammation seen on colonoscopy Stool test positive 250 50 Stool test negative 25 300 Which of the following is the sensitivity for this stool test? A 92% B 91% c 86% D 83% e 8% ee Blog About Pastest Contact Us Help © Pastest 2023 Question 8 of 100 ‘A new tests under development forthe detection af inflammatory bowel disease, The performance characteristics of the test are shown below: 50 300 c A 92% BE c 86% D 83% £ 8% Explanation Ad 2 sitivity, calculated by dividing the true positives by the true positives plu 0 + 25) = 91%. he false negatives, ctve value, calculated by dividing the true negatives by the true negat false negatives, or 300/(300 + 25) = 92%. plus the c 86% This isthe specifity, calculated by dlviding the true negatives by the true negatives plus the false positives, .r300)/(300 + 50) = 8696, D 83% ‘Tris is the positive predictive value, calculated by aviding positives, o 250 /[250 + 50) = 8396 true positives by the true postive plus false £ aw This is one minus the negative predictive value. Itis chance that a tue case willbe missed. known asthe false omi n rate or the percentage Blog About Pa Contact Us Help © Pastest 20 9 a @ 3 Qa 5 96 q 8 Answered Flagged oO oO oO oO Oo Oo Oo 0 aerey Bory byl rao Question 9 of 100 74-year-old man is reviewed in the Haematology Clinic after multiple transient ischaemic attacks (TIAs) in the last three months. On further questioning, it transpires he has suffered multiple headaches over the last five months. He has hypertension and suffered an inferior myocardial infarction some four years earlier. His blood pressure is 155/92 mmHg and his pulse is 84 bpm and regular. He has no residual neurological deficit. You note a petechial rash on skin examination, Investigations: Investigation Result Normal value Haemoglobin (Hb) 159 oft 135-175 glk White cell count (WCC) 71x 10% 4-11 x 10% Platelets (PLT) 1250 x 10%. 150-400 x 10% Sodium (Na*) 140 mmol/L 135-145 mmolft Potassium (K*) 5.2mmollt 3.5-5.0 mmol Creatinine (Cr) 112 pmol 50-120 pmol/l Which of the following is the most appropriate intervention? A Aspirin B Hydroxyurea C _Interferon-alpha D _Phosphorus-32 E Splenectomy Previous Question Save and Leave End Session Skip Question Blog About Pastest ContactUs Help © Pastest 2023 See teeta aste) eee {A74-year-old mans reviewed in the Haematology Clinic aT - the last three months. On further questioning, transpires he has (RRHORREE He has hypertension and suffered an inferior myocardiat infarction some four years earlier. His, Poor Responses % blood pressure is 155/92 mmHg and his pulse Is 84 bom and regular. He has no residual neurological dec. ‘You note 3 RRERHINARH on skin Investigation Result Normal value < Haemoglobin (Hb) 159 gt 135-175 git ° White cll count (WCC) 7110 411 «10 Sedum (83%) 40 mmott 135-145 melt & Answered Ragaed (Potassium 4) ‘S2mmolt 35-50 molt ica a Creatine (C2) 112 pmol 50-120 matt @ 0 Wich ofthe felowing is the most appropri intervention? % 5 a 0 9s 5 Sy a % 0 Bee 7 . a eee oe a > Photphors-32 ° & | ere st Teno al Explanation ° ° ( Hydroxyurea isthe inital intervention of choice for elderly patients with primary thrombacytosis. Patients at highest risk of thromboembolic events are those over 60 and those with PLT counts in excoss of 1500 Hydroxyurea acts as a false metabolite and is highly effective in reducing PLT counts. Ideally, patients should be treated to reduce PLT counts to botow a target of 450. A Aspirin ‘Tris patient has primary thrombocytosis and has suffered a number of TIAs as a result Hei, therefore considered high risk, especially given the PLT count is markedly elevated at 1250. As such, low-do is not appropriate and mare aggressive therapy with an agent such 2s hydroxyurea is required aspirin © Interferon-aipha Interferon-alpha isthe intervention of choice in younger patients. tis a biological response modifier. tis safe {or use in pregnancy because it does nat cross the placenta and there is no known teratogenicity. One cisadvantageis that PLT counts tend to rebound once therapy is discontinued D_Phosphorus-32 Phosphorus-32 is reserved for patients who fal to see a reduction in PLT count when treated with conventional agents such as hydroxyurea, anagrelide or busulfan. One risk of phosphorus-32 therapy is that ican accelerate leukaemic transformation to acute myeloid leukaemia, which is seen in 0.5-59 of patients with pximary thrombocytosis overall. E Splenectomy In contrast to idiopathic thrombocytopenic purpura, splenectomy isnot of value inthe treatment of primary ‘thrombocytasis. t may increace the risk of both thrambosis and haemorrhage. Previous Question ag Question Feedback nd Review Blog About Pastest Contact Us Help 4 © Pastest 2023 OES Bory Jay Pre Question 10 of 10 72-year-old man is referred by his General Practitioner for investigation of intermittent dysphagia for solids and liquids over the past nine months. He also has retrosternal pain, which he says is relieved by regurgitating food. He also reports a cough at night and has had one episode of right lower lobe pneumonia in the past year. He has lost no weight. His blood pressure is 148/84 mmHg and his pulse is 72 bpm and regular. He has mild tenderness in the epigastrium on firm palpation Investigation Result Normal value Haemoglobin (Hb) 139 git 135-175 gil White cell count (WCC) 72x10" 4-11 x 10% Platelets (PLT) 309 x 10%/1 150-400 x 10° Sodium (Na*) 140 mmol/l 135-145 mmol/l Potassium (K*) 4.1 mmoUl 3.5-5.0 mmol Creatinine (Cr) 117 pmott 50-120 pmolt Erythrocyte sedimentation rate (ESR) 8 mm/hour < 10 mm/hour A barium swallow test was performed. Results show dilated oesophagus with bird's beak appearance. Which of the following is the most effective intervention? A Amlodipine B Botulinum toxin C Isosorbide dinitrate D__ Nissen’s fundoplication E Pneumatic dilatation Previous Question Save and Leave End Session Skip Question Blog About Pastest ContactUs Help © Pastest 2023 Seo (Netcare sT000 ere (A72-year-old man is referred by his General Practitioner for ivestpation of B= over the past rine months. He also He alo reports 3 ESURISRIGRE 2nd nas ha eer Responses 9 in the past year. He has lost no weight His blood pressure is 148/84 mmHg and his pulse is 72 bpm and regular, He has mild tendemess in the epigastrium on fim palpation, A Investigation Result Normal value Haemoglobin Hb) 139 gf 135-175 9h . White cell count (WCC) 72%10% 41110 . a Platolets (PLT) 309% 10% 160-400 10%" ae Sodium (Na") 140 mmovt 135-145 mmol Potassium (K") 4.1 mmol 35-50 mmout ices Creatinine (CA 117 umolt 50-120 pout cad a Erythrocyte sedimentation rate (ESR) 8 mmMhour 10mmihour @ D ‘Abbarium swallow test was performed, Results show dilated oesophagus with bird's beak appearance, as a Which of the following is the most effective intervention? ry ' Your answer was incorrect 8 Oo ry 0 A Amlodipine q 0 B Botulinum toxin 8 i ° C Isosorbide dinitrate ° D Nissen’ fundoplication Je [rmmeccnnm Explanation * B a : Pneumatic clilatation is the initial intervention of choice for achalasia the diagnosis here. Achalasa is characterised by intermittent dysphagia to liquids or solids ands related to spasm ofthe lower oesophageal sphincter. The lack of weight loss and the range of normal blood tests count against an underlying malignancy. Between 40% and 85% of patients with achalasia have a successful dilatation, with low rates of perforation (< 10%), when treated by an experienced endoscopist. Where patients fall to gain benefit fram dilatation, surgery isthe next step, depending on functional status. A Amlodipine Calcium channel antagonists are effective in some patients who have achalasia, the likely diagnosis here Unfortunately, only around 1096 of patients gain some symptom relief from these agents, For this reason, they are generally reserved for patients who fal to respond to other intervention. B Botulinum toxin Up to 85% of patients gain some short-term rele from injection of botulinum toxin, although atthe 2-year time point, only 303% have stil gained some benefit. Botulinum toxin's, therefore, usualy reserved for patients who are too unwell to undergo pneumatic cittation or surgery. C_leosorbide dinitrate Like calcium channel antagonists, nitrates can be used for the treatment of achalasia, although only a small, percentage of patients respond, and over time, tachyphylaxis tothe effects of nitrates is seen D__Nissen’s fundoplication Nissen’ fundoplication is the intervention of choice for hiatus hernia It isnot the intervention of choice for achalasia — Heller's myotomy i. can be performed via the laparoscope. The muscle fibres of the lower ‘oesophagus are divided longitudinally for about 5 em, about 1.5 em above the entrance to the stomach. More ‘han 85% of patients can benefit from the intervention, Previous Question ag Question Feedback nd Review Blog About Pastest Contact Us Help a © Pastest 2023 Question 11 of 10 72-year-old man presents to the Neurology Clinic with his wife. Over the past few months, he has become increasingly unsteady on his feet, suffers from dizziness, particularly when he stands up and has increasing problems with urinary incontinence. His General Practitioner has trialled levodopa to see if it improved his movement disorder, but it appears to have had little impact on his symptoms. He has an ataxic gait as he walks into the consulting room. His lying blood pressure is 155/90 mmHg which drops to 115/75 mmHg on standing, You note increased tone and rigidity on neurological assessment, more marked on the left than on the right. There is a very minor tremor. Which of the following is the most likely diagnosis? ‘A Idiopathic intracranial hypertension B Idiopathic Parkinson's disease C Lewy body dementia D — Multisystem atrophy E Normal pressure hydrocephalus Previous Question Save and Leave End Session Skip Question Blog About Pastest ContactUs Help © Pastest 2023 Question 11 of 100 {A72-year-old man presents to the Neurology linc with hs wif, Over the past few months, he has become increasingly unsteady on his fect, sues from diaziness, particulary when he stands up and has increasing problems with urinary incontinence His Genera Practoner has 10 see ftimproved his movement disorder, butt appears to have had He has an BEBREWAR as ne walks into the consulting room. His lying blood pressures 155/90 mmig which drops to 135/75 mmtig on stancing. You note| lon neurological assessment, more marked on the left than on the right. There ie avery Which ofthe following is the most likely diagnosis? A. Idiopathic intracranial hypertension Idiopathic Parkinson's disease C__ Lewy body dementia en E Normal pressure hydrocephalus Explanation * “The features of ataxia, muscular rigidty with tremor and marked autonomic dysfunction fit well with a iagnosis of multisystem atrophy. Movement disorder often responds very poorly to levodopa, as seen here ‘Symptoms progress rapidly and include a significantly increased falls sk, with progressive cognitive impairment. Mean survivals between six and nine ye A Idiopathic intracranial hypertension Idiopathic intracranial hypertension occurs primary in obese Individuals and is seen more commonty in younger women. t presents with headaches which are usually worse in the morning and when patients strain, Staining may be associated with blurring of vision or loss of colour vision. Weightloss is the most important tong-term intervention, Idiopathic Parkinson's disease Idiopathic Parkinson's disease is incorrect because the autonomic neuropathy features seen here would be ‘unexpected and a more marked tremor is usually seen, A better response to levodopa would also be expected, Lewy body dementia Lewy body dementia is associated with cognitive impairment and often with visual hallucinations, which pre date the typi hallucinations is dificult, with tractional dopamine antagonists best avoided. Clozapine isthe preferred long-term intervention for controling psychosis 1, movement disorder seen with idiopathic Parkinson's disease. Management ofthe E Normal pressure hydrocephalus Normal pressure hydrocephalus is associated withthe triad of movement disorder, urinary incontinence and ‘memory loss. t does not Fit withthe prominent features of autonomic neuropathy seen here Previous Question ag Question Feedback nd Review Blog About Pastest Contact Us Help © Pastest 2023 Q Answered Flagged a Oo @ Oo @ Oo Cr Oo 5 Oo 96 Oo a Oo 98 Og — © a ae ey Question 12 of 10 A 29-year-old man presents to the Rheumatology Clinic with recurrent oral and genital ulceration and superficial thrombophlebitis affecting his left lower limb. He has also had two episodes of anterior uveitis in the last six months. Examination reveals multiple aphthous ulcers and a rash on both shins, consistent with erythema nodosum Investigation Result Haemoglobin (Hb) 101 g/t White cell count (WCC) 13.2 x 10% Platelets (PLT) 393 x 10% Sodium (Na°) 144 mmol Potassium (K*) 3.7 mmol Creatinine (Cr) 118 pmol/L C-reactive protein (CRP) 85 mg/l Erythrocyte sedimentation rate (ESR) 54 mm/hour Which of the following is the most appropriate intervention? A Azathioprine Cyclophosphamide Etanercept Prednisolone Rituximab Normal value 135-175 git 4-11 x 10% 150-400 x 10% 135-145 mmol/L 3.5-5.0 mmol/l 50-120 pmol/L <10 mg/l <10 mm/hour Previous Question Save and Leave End Session Blog About Pastest ContactUs Help © Pastest 2023 Skip Question Question 12 of 100 Cog ‘A 29-year-old man presents tothe Rheumatology Clinic with recurrent oral and genital ulceration and superficial thrombophlebitis affecting his eft lower limb, He has also had two episodes of anterior uveitis in the last six months, Examination reveals multiple aphthous ulcers and a rash on both shins, consistent with Peer Responses % cexythema nodosum, Investigation Result Normat value somat 135-175 git . 1B2"10% 441 x10" a ST Qi vO roc sceertatien tS) Stmmour commu “ . 3 oO Caer creeper reer a ao 5 UL yee es a q Oo o a ° c Etanercent — ie Rituximab a Explanation 2 oR 4 This patient has evidence of oral and genital ulceration, anterior uveitis and thrombosis. Coupled with ‘erythema nodosum found on examination and raised inflammatory markers, this Mts witha diagnosis of BBenot’scisease. Given there is eye involvement, systemic therapy with corticosteroids is indicated. Second: line options for therapy include cyclophosphamide, azathioprine and etanercept A Azathioprine ‘Azathioprine is indicated for long-term treatment of Behcet's disease, the Ukely diagnosis here, tis used in ‘the event that the patient's symptoms are not contclled on cortcostercids alone or itis not possible to completely wean off prednisolone B Cyclophosphamide Cyclophosphamide is indicated for patients with Behet's disease who have centra nervous system involvement where a more aggressive intervention is required. Cyclophosphamide is usually combined with corticosteroids, C Etanercent Etanerceptis an effective second-line alternative to azathioprine and cyclophosphamide in patients with BBence’s disease, Its associated with a reduced relapse rate vs prednisolone combined with conventional mmunosuppressive agents Rituximab Rituximab has been used where symptoms of severe Behcet's disease fall to respond to a standard range of interventions, including prednisolone with a second-line agent such as cyclophosphamide or azathioprine Previous Question ag Question Feedback nd Review Blog About Pastest Contact Us Help Q © Pastest 2023 Sy Br Joy Yr ae Question 13 of 10 A 47-year-old woman is referred to the Cardiology Clinic by her General Practitioner (GP). She has suffered 2-3 episodes of syncope over the past three months and feels constantly short of breath. An electrocardiogram (ECG) recorder she bought for her smartphone has demonstrated periods of paroxysmal atrial fibrillation (AF). Her blood pressure is 132/84 mmHg and her pulse is 80 bpm and regular. She has fixed splitting of the second heart sound and a mid-systolic murmur loudest at the second left intercostal space. Her chest is clear and there is mild pitting oedema of both ankles. Investigation Result Normal value Haemoglobin (Hb) 135 g/t 115-160 g/l White cell count (WCC) 8.0x 10%. 4-11 x 10% Platelets (PLT) 203 x 10%. 150-400 x 10% Sodium (Na*) 144 mmol/l 135-145 mmol/l Potassium (K") 4.9 mmol 3.5-5.0 mmol Creatinine (Cr) 110 pmol/L 50-120 pmolft Which of the following is the most likely cause of her symptoms? A Ostium secundum atrial septal defect B Brugada syndrome Hypertrophic cardiomyopathy D Rheumatic heart disease E Ventricular septal defect Previous Question Save and Leave End Session Skip Question Blog About Pastest ContactUs Help © Pastest 2023 Pee elsy (ia Question 13 of 100 A.47-year-old woman is referred to the Cardiology Clinic by her General Practitioner (GP). She has suffered Her chests clear and there is mild pitting oedema of bath ankles Investigation Result Normal value . Haemoglobin (Hb) 1359 115-160 gf ° White cell count (WCC) a0 x10% 4at e107 c Platelets (PLT) 203 x 10 150-400 x 10°" fe ae see ee Sodium (Na") 144 mmout 135-145 mmol Q Answered Flagged Potassium (K") 49 mmol 35-50 mmol Cs o Creatinine (Cr 120 pout 50-120 uot a ; Which of the following is the most likely cause of her symptoms? 3 o 98 0 CoE d rd Oo B Brugada syndrome @ ri C__ Hypertrophic cardiomyopathy 8 O69 D_Rheumatic heart disease E Ventricular septal defect Explanation o «| Ee ‘Tris woman's presentation with syncopal episodes, paroxysmal AF. ankle swelling, a split-second heart sound and 2 mi-systolic murmur (Ukely due to increased flow across the pulmonary valve) is consistent with an atrial septal defect and a left-to-right shunt. Electracardiogram (ECG), chest X-ray and tranzoesophageat echocardiography (ECHO) ate the initial. interventions of choice She should be considered for @ procedure to close the atiat septal defect. which can be percutaneous or via surgical approach B Brugada syndrome Brugacla syndrome is characterised by a conduction abnormality which results in increased risk of sudden death. Unike hypertrophic cardiomyopathy (HCM) or long QT syndrome, these episodes are not necessarily related to physical exertion. The characteristic abnormality is ST elevation in leads V1 to V3, with aright bundle branch black (RBBB) appearance and a number of sodium channel mutations that have been dented and are linked tothe syndrome, The hearts structurally normal; therefre, the patient would have a normal physical examination. C Hypertrophic cardiomyopathy Classical examination findings in HEM include a forceful apex beat with a double impulse if theres also left ventricular outflow tract obstruction. A late ejection systolic murmur may also be heard, which is augmented by standing. This murmur can be the result of systolic anterior motion of the mitral valve in severe cases of the disease. D Rheumatic heart disease Rheumatic heart disease does not ft with the clinical findings here where mitral stenosis is often seen, characterised by a rumbling low-pitched diastolic murmur. Changing patterns of streptococcal infection now ‘mean thatthe incidence of rheumatic fever has fallen significantly over past decades and, asa consequence, the incidence of cardiovascular complications has also decreased, E Ventricular septal defect A large ventricular septal defect (VSD) would not present for the fist time ina patient in her ith decade. In ‘a patient of this age, a small VSD might present with a systolic murmur best heard at the left sternal edge, “The noise ofthe second heart sound may be obscured by the murmur. Previous Question ag Question Feedback end Review Blog About Pastest ContactUs Help Q © Pastest 2023 x SEN Sy BA axl FF aso Question 14 of 10 A 32-year-old man is reviewed in the Emergency Department following a collapse with severe headache and palpitations after chasing after a bag snatcher in the street. On further questioning, he admits to a number of episodes over the past six months when he has suffered from palpitations and headaches, some of which have lasted for a few hours. His blood pressure is 155/95 mmHg and his pulse is 92 bpm and regular. Heart sounds are normal and his chest is clear. His abdomen is soft and non-tender. Body mass index is 23 kg/m2 Investigation Result Normal value Haemoglobin (Hb) 144 git 135-175 git White cell count (WCC) 78x 10% 4-11 x 10% Platelets (PLT) 317 x 10% 150-400 x 10%/1 Sodium (Na*) 143 mmol/L 135-145 mmol/l Potassium (K*) 3.7 mmol/l 3.5-5.0 mmol/L Creatinine (Cr) 118 pmol 50-120 umol/t Bicarbonate (HCO3") 27 mmolht 23-30 mmol/L Glucose 10.1 mmol < 11.1 mmol Which of the following is the most useful investigation? ‘A 24-hour urine collection for catecholamines B 24-hour urine collection for cortisol C 24-hour urine collection for 5-hydroxyindoleacetic acid (HIAA) D Echocardiogram — Overnight urine for albumin : creatinine ratio Previous Question Save and Leave End Session Skip Question Blog About Pastest ContactUs Help © Pastest 2023 Pee elsy (ia Question 14 of 100 ‘A 32-year-old man is reviewed in the Emergency Department following 2 SSS ih Sve RESEERS On further questioning, he admits to lof which have lasted fora few hours. His blood pressure is 155/95 mmHg and his pulse is 92 bpm and regular, Heart sounds are normal and his chest i clear. His abdomen is soft and non-tender. Body mass CY index is 23 kg/m Investigation Resutt Normal value c Haemoglobin (ib) ae oh 135-1759 b Wiite cl count (WCC) 72x 10% sare t0% . Platelets (PLT) 317 x 10% 150-400 x 10% fm go se 980 Sodium (No) 143 mmott 135-145 molt et Potassium i) 3.7 mmott 35-50 mmott Gil ri Creatinine (Cx 118 uot 50-120 umott ci ’ mot = = Bicarbonate (HCO 27 molt 23-30 mmolt @ Oo Glucose 10.1 amoiA <1. mmott a 0 Which ofthe following isthe most useful investigation? 5 UD a6 0 Sa Ce on oO ae 0 B 24-hour urine collection for cortisol ° 24-hour urine coleton fr S-hydronyndoleactc acid (HAA) ° a D Echocardiogram E Overnight rine for albumin creatinine rato ® @ 4% Explanation * ‘Tris patient has paroxysms of hypertension and tachycardia with headaches. loods are relatively normal, apart from the glucose level towards the upper end of the normal range. Ths constellation of symptoms raises the possibilty of a phacochromocytoma. A 24-hour urine collection for catecholamines isthe most appropriate next step. Computed tomography (CT) isthe initial scanning modality of choice. although ‘magnetic resonance imaging (MRI) andlor metaiodobenzylquanidine MIBG) scanning may be required for tumour localisation. Full alpha blockade with phenoxybenzamine is crucial before surgical excision is attempted See ee 8 24-hour urine calection for cortisol {A 24-hour urine collection for cortisol isthe initials ening test of choice for Cushing syndrome, Cushing syndrome ie associates with weight gain, hypertension, hyperglycaemia and hypokalaemic metabolic alkalosis. The blood picture and body weight ae inconsistent with a Cushing syndrome diagnosis. C 24-hour urine collection for S-hydroxyindoleacetic acid (HAA) Urine collection for 5:HIAA isthe screening test of choice for carcinoid syndrome, Carcinoid syndrome is characterised by episodes of facial flushing, palpitations and diarrhoea, It does not fit wth the clinical picture ‘seen inthis patient. Carcinoid syndrome is also seen more often inthe elderly population D Echocardiogram {An echocardiogram will not reveal the underlying cause ofthis patient's hypertension and tachycardia, but it may reveal evidence of left ventricular hypertrophy, cantingent upon the duration of his symptoms, Urinary catecholamines are preferred because these will provide evidence of an underlying phaeachromacytoma, £ Overnight urine for albumin: creatinine ratio Urine albumin excretion from an overnight collection is useful in determining the prognosis of proteinurc renal disease. It's not of prognostic value inthe evaluation of phacochromacytoma, Proteinuria may well be seen on urine testing as a result of chronically elevated blood pressure Previous Question ag Question Feedback nd Review Blog About Pastest Contact Us Help a © Pastest 2023 = BENS Bors Jay rr ae Question 15 of 10 A 28-year-old woman presents to the Emergency Department complaining of vague abdominal pains over the past few weeks. She is very anxious and is concerned that she may have cancer. On reviewing her notes, you find that over the last year, she has had two colonoscopies, two upper gastrointestinal endoscopies and an abdominal computed tomography (CT), all of which have been normal. She tells you that her father died of bowel cancer aged 63 years. Physical examination is entirely normal Investigations: Investigation Result Normal value Haemoglobin (Hb) 132 g/l 115-155 g/l White cell count (WCC) 74x 10% 4-11 x 10% Platelets (PLT) 320 x 10% 150-400 x 10° Sodium (Na*) 143 mmol/l 135-145 mmol/L Potassium (K*) 4.2 mmol 3.5-5.0 mmol Creatinine (Cr) 90 umolt 50-120 pmolt Erythrocyte sedimentation rate (ESR) 9 mm/hour < 10 mm/hour Which of the following is the most likely diagnosis? A Crohn's disease B Coeliac disease C Depression D Illness anxiety disorder — — Munchausen syndrome by proxy Previous Question Save and Leave End Session Skip Question Blog About Pastest ContactUs Help © Pastest 2023 See Neer BEL aT0l0 ere ‘A 28-year-old woman presents tothe Emergency Department complaining of JG U=EaGREUBERE over 5 the past few weeks. She is very anxious and is concerned that she may have cancer. On reviewing her notes you find that Peer Responses % ‘She tells you that her father died ‘of bowel cancer aged 63 years. Physical examination is entirely normal 4 Investigations: . Haemoglobin (Hb) 132g 115-155 oft os ‘White cell count (WCC) 74x10 4-11x 10% ‘Sodium (Na*) 143 mmoUt 135-145 mmol a Lioheofonit tt lea Potassium (K") 4.2 mmol 3.5-5.0 mmol qi vO a 0 q Oo © Depression a ° c Munchausen syndrome by proxy Explanation o ® & 4 ‘This patient is extremely anxious and believes that she may have cancer despite a large number of norma investigations over the past year and normal blood workup. Itis tkely thatthe trigger forherilness anxiety disorder isthe death of he father from bowel cancer. Cognitive behavioural therapy isthe intial intervention of cholee, Selective serotonin reuptake inhibitors may be considered for some ps psychological intervention. ents who are resistant to A Crohn's disease Inflammatory bowel disease can present with abdominal pain, although it does not fit with the normal blood picture seen here, Te lack of anaemia and no signs of inflammation favour e functional disorder. 8 Coelise disease Coeliac disease is associated with anaemia and signs and symptoms of malabsorption; features not seen here. tis also often associated with dermatitis herpetiformis, an intensely itchy rash with papul vesicles, which particulary affects the buttocks and the top ofthe thighs. and C Depression Although ansiety often coexists with depression, we have no features of depression per se described here For this reason, illness anxiety disorder, rather than depression, is the more likely diagnosis here, E —_Munchausen syndrome by proxy "Munchausen syndrome by proxy involves a caregiver making up an illness in, or causing injury to, a person Under their care such as a child or an elderty adit. It does nat fit with the picture seen here. Previous Question ag Question Feedback nd Review Blog About Pastest Contact Us Help 4 © Pastest 2023 = BENS Bors Jey! tr aod Question 16 of 10 ‘A59-year-old man who smokes 30 cigarettes per day presents to the clinic for review. He has been referred because of a cough, worsening glycaemic control and hypertension. He takes a range of medications for hypertension and diabetes mellitus. His blood pressure is 155/92 mmHg and his pulse is 84 bpm and regular. He has a wheeze on auscultation and quiet breath sounds consistent with chronic obstructive pulmonary disease. His body mass index (BMI) is 28 kg/m?. Investigations: Investigation Result Normal value Haemoglobin (Hb) 122 g/t 135-175 gil White cell count (WCC) 10.9 x 10% 4-11x 10% Platelets (PLT) 391 x 10% 150-400 x 10% Sodium (Na*) 144 mmol/L 135-145 mmollt Potassium (K*) 3.4 mmol/l 3.5-5.0 mmol/L Creatinine (Cr) 132 pmol 50-120 umol/t Bicarbonate 34 mmol/l 23-30 mmol/L Fasting glucose 13.2 mmol <7.0 mmol/t Chest X-ray (CXR) Left hilar mass Which of the following is the most likely cause of his symptoms? ‘A Adenocarcinoma of the bronchus B Bronchial carcinoid C Sarcoidosis D Small cell carcinoma of the bronchus — Squamous cell carcinoma of the bronchus Previous Question Save and Leave End Session Skip Question Blog About Pastest ContactUs Help © Pastest 2023 Question 16 of 100 ‘A.59-year-old man who smokes 30 cigarettes per day presents tothe clinic for review. He has been referred because ofa cough, worsening glycaemic controt and hypertension. He takes a range of medications for hypertension and diabetes malts. His blood pressure is 155/92 mmHg and his pulse is 84 bpm and regular. He has a wheeze on auscultation and quiet breath sounds consistent with chronic obstructive pulmonary disease, His body mass index (BM) is 28 kai’ Investigations: Investigation Normal value I White cell count (WCC) 10910" 411% 10% Platelets (PLT) 391% 10 150-400 x 1071 Sodium (Na") 144 mmol 135-145 mmout Which ofthe following is the most likely cause of his symptoms? A Adenocarcinoma ofthe bronchus B Bronchial carcinoid C Sarcoidosis om uel ener eEERES E Squamous cell carcinoma of the bronchus Explanation 2 Peace ‘Tris patient has a raised bicarbonate with hypertension and diabetes mellitus. Coupled with a hilar mass on ‘the CXR, this is suspicious of Cushing syndrome as a result of ectopic adrenocortcotrophic hormone (ACTH) secretion, from an underlying smell cell carcinoma ofthe bronchus, Symptoms of Cushing syndrome can resolve with treatment of the underlying primary cancer. Small cell ung eancer may also present with ssmndrome of inappropriate antiluretic hormone secretion A Adenocarcinoma ofthe bronchus ‘Adenocarcinomas do not occur asa result of smoking and are not usually hormone-secreting. This man's symptoms are consistent with a tumour producing ectopic ACTH, resulting in Cushing syndrome. They are Usually seen as a nodular lesion on the CXR, B Bronchial carcinoid Bronchial carcinoid is more commanly seen in younger patients and is not associated with smoking. They are dlscrete vascular lesions that can present with haemoptysis oF cause recurrent pneumonia due to bronchial obstruction. Sarcoidosis Sarcoidosis results in bilateral lymphadenopathy on X-ray and patients may complain of @ dry cough. Some patients with sarcoidosis may also present with symptoms of hypercalcaemia. tis not associated with the development of ectopic ACTH production and Cushing syndrome. E Squamous cell carcinoma of the bronchus ‘Squamous cal carcinoma ofthe bronchus Is not associated with Cushing syndrome. Itis associated with hypercatcaemia because of tumour production of parathyroid hormone-related peptide, which can lead to ‘typical symptoms of hypercalcaemia, including polyuria, plydinsia, nausea, vomiting andl confusion. ag Question Feedback nd Review Blog About Pastest Contact Us Help © Pastest 2023 Q Answered Flagged a Oo @ ( @ Oo 4 ( 5 Oo 96 Oo a Oo ae ( ° — ° a ® 6 4% Question 17 of 10 You are investigating the potential to treat patients with non-alcoholic steatohepatitis using a thyroid hormone receptor agonist. Where do drugs need to be targeted to achieve receptor agonism? A Cell surface B Endoplasmic reticulum C Golgi body D Mitochondria — Nucleus Previous Question Save and Leave End Session Skip Question Blog About Pastest ContactUs Help © Pastest 2023 Question 17 of 100 ere You are investigating the potential to treat patients with non-alcoholic steatohepatitis using Where do drugs need tobe targeted to achieve receptor agonism? Poor Responses % A Call surface 8 Endoplasmic reticulum as © Goi body Answered Flagged D Mitochondria = ry | * . 3 Oo Explanation o co 0 | _ ia “The thyroid hormone receptors nuclear receptor that acts via a genomic and a non-genomic pathway. The a6 o non-genomic pathways activated when T3 binds tothe thyroid hormone receptor, which then leads to an increase in clic guanosine monophosphate (GMP) In tun this leads t activation of cyclic GMP-dependent a Oo protein krase, which has a postive elect on cell vit. The genomic pathway of thyroid hormene receptor activation regulates gene transcription. The receptor can regulate tanscription as a monomer, but italso acts ae De in tandem with ether nucear receptors Such asthe retinoid X receptor A Call surface ° eal surface receptors are defined into tvee main types yg a + ion channet-tinked receptors suchas those targcted by neurotransmiters a + enzymetinked receptors, which are usualy potsn kinases or associated with poten kinases + G-proten-coupled receptors, which interact with integral membrane proteins that possess seven B ‘transmembrane helices, then mediating downstream receptor signalling processes, B Endoplasmic reticulum Fibosomes bind tothe rough endoplasmic reticulum and ar involved in protein synthesis. A ribosome only binds tothe rough endoplasmic reticulum when a specific protein-nuceic acid complex forms in the cytosol € Golgi body “The Golgi body is involved inthe processing and packaging of proteins and tpid molecules. The Golgi body Is not a usual target for hormones or for drugs. D Mitochondria -honetia 3r23 drug target to reduce ageing and improve energy metabolism within the cell. Their function is nat affected directly by thyroid hormone, Examples of agents that improve mitochondrial function include sodium-glucase co-transporter 2 (SGLT-2) inhibitors which stimulate gluconeogenesis. Question ag Question Feedback nd Review Blog About Pastest ContactUs Help © Pastest 2023 Question 18 of 10 A 29-year-old man presents to the clinic for review with his child who is eight. He has recurrent angioedema, which is unresponsive to antihistamines and associated with both low C4 and low C1 esterase inhibitor levels. He is concerned that his children may have inherited the condition. Which of the following is the percentage chance of his child suffering from the condition? A 100% B 50% ae 25% D <1% E 0% ee Previous Question Save and Leave End Session Skip Question Blog About Pastest ContactUs Help © Pastest 2023 Question 18 of 100 {A 29-year-old man presents to the clinic for review with his He has hich is unresponsive to antihistamines and associated with both TERRE Heis concerned that his ehitdron may have inherited the condition Which ofthe following is the percentage chance of his child suffering from the condition? A 100% a c 25% D «1% E O% Explanation ° The patient has hereditary angioedema, which is inherited in an autosomal dominant fashion. The patient has a 50% chance of passing the affected gene an te his children, Its a rare concition, with a globat incidence of 1 in 50 000. The onset of symptoms of hereditary angioedema usualy corresponds with puberty. A numberof interventions are available including a C1 inhibitor and a bradykinin receptor inhibitor called fativant A 100% Tis patient has hereditary angioedema, which is inherited in an autosomal dominant fashion. To have a 100% chance of passing on the condition to his children, he would have to be homazygous for the aff ‘Gene, This is vanishingly rare ~a literature search revealed one reported case of homozygosity from 2006, cc 25% This isthe chance ofa patient inheriting an autascmal recessive condition when both parents are carriers of the affected gene Dim ‘This is incorrect. The incidence of hereditary angio-edema is well below 1%, at round L in 50.000. E 0% Hereditary angio-cedems is autosomal dominant: hence, the percentag concition is 5096 1ance of a chil inheriting the Previous Question ag Question Feedback nd Review Blog About Pas Contact Us Help © Pastest 20 Q Answered Flagged Qu Oo @ ( 3 Oo 4 Oo 5 Oo 96 Oo Cy Oo a8 Do ° a Question 19 of 10 ‘An 84-year-old woman who has been found unconscious presents to the Orthopaedic Ward. She has continued to receive her twice-daily mixed insulin, but her evening meal was found untouched at the side of the bed. A finger prick blood glucose is measured at 1.4 mmol/l. She mumbles when vigorously shaken. Which of the following is the most appropriate next step in this patient? A 100 ml of 20% glucose over ten minutes B 500 mlof 5% dextrose over 30 minutes C 1 litre of 5% dextrose over 30 minutes D 1 mg of intramuscular (IM) glucagon E Oral dextrose tablets Previous Question Save and Leave End Session Skip Question Blog About Pastest ContactUs Help © Pastest 2023 Question 19 of 100 ‘An 84-year-old woman who hi en (SURES presents tothe Orthopaedic Ward. She has continued to receive her twice-dily mixed insulin, but her evening meal was found untouched atthe side of {he boc AIAG bd GUESS MaSUAIAELAMIMGIA She mumbles when vigorously shaken Which of the following is the most appropriate next step in this patient? eee te B50 ml of 596 dextrose over 30 minutes CL ttre of 5% dextrase aver 30 minutes DL mgof intramuscular (IM) glucagon E Oral dextrose tablets Explanation o BE ‘This patient has life-threatening hypoakycaemia, most likely as a result of continued administration of insulin without maintaining adequate calorie intake. significant glucose load is therefore, requied over a short period of time. This ean be delivered as 100 ml af 20% glucase over ten minutes, or as 200 ml of 10% alucose. B 500 ml of 596 dextrose over 30 minutes eel eee ete ee et eee eee et ee appropriate option here because it will not significantly elevate bload glucose, Smaller volumes of more concentrated glucose are required CC Llitre of 5% dextrose over 30 minutes Although 1 live of 5% dextrose represents a significant volume of fluid it does not contain a significant load of carbohydrate to reverse hypoglycaemia. As suc, itis not an appropriate intervention here. D__Lmg of intramuscular (IM) glucagon Intramuscular (IM) glucagon is @ potential option in the event that vascular access cannot be obtained immediatly. Glucagon delivered intravenously is more effective, butt is associated with a significant risk of nausea and vomiting E Oral dextrose tablets Given this patients level of consciousness, ora treatments for hypoglycaemia, such as dextrose tablets, are not an appropriate intervention. When patients are sufciently conscious to take calories orally, short-acting carbohydrates, such as dextrose tablets or orange juice, should be given together with more stow releasing sugars such as digestive biscuits or toast Previous Question ag Question Feedback nd Review Blog About Pastest Contact Us Help © Pastest 2023 Q a 2 3 Qs Os ry q 8 Preealvenen Answered Flagged 0 ° a Sy Pvt byl ted Question 20 of 10: A 54-year-old man is reviewed on the Medical Admissions Unit following admission with a stroke. He is a non-smoker with no significant medical illnesses. His blood pressure is 152/94 mmHg and his pulse is 80 bpm and regular. He seems to have rather plethoric facies. Investigations: Investigation Result Normal value Haemoglobin (Hb) 201 git 135-175 g/l White cell count (WCC) 138 x 10% 4-11 x 10% Platelets (PLT) 495 x 10% 150-400 x 10°/t Sodium (Na*) 140 mmol/t 135-145 mmol/L Potassium (K*) 5.1 mmol/l 3.5-5.0 mmol/L Creatinine (Cr) 129 molt 50-120 umoU/t Which of the following mutations is he most likely to be carrying? A BRCA2 B c2e2y C Delta-F508 D JAK2 E P53 Previous Question Save and Leave End Session Skip Question Blog About Pastest ContactUs Help © Pastest 2023 Question 20 of 100 Cog {A 54year-old man is reviewed on the Medical Admissions Unit following | bpm and regular, He seems to have Poor Responses % Investigations: Investipaton Fest Normal value : Haemoatoin (6) 201 ot 13547598 C Write eet count (WC) 138%10% aan a0% cere] Praetets (PT) 95109 150-400% 10% . iiGddddddae Sodium Ne") 140 mmott 135-145 melt etossum >) 51 mmout 35-50mmotA Q@ Mam ged Creatinine 129 pmolt 50-120 ald a 5 ‘Which of the following mutations is he most likely to be carrying? Q2 UD @ O a 0 A aRcaz 8 UL 8 conv w oO CC Delta-F508 a 0 oe a a Explanation o This patient has primary polycythaemia, as evidenced by elevated Hb, WCC and PLT count. Mutation in JAK2 is considered a major criterion for the diagnosis of polycythaemia. These include the JAK2 617V F mutation or other functionally similar mutations such as the exon 12 mutation of /AK2. The mutations lead to JAK2 enzyme system being continuously switched on, which leads to uncontrolled blood cell production. A BRCAZ [BRCAZis 3 tumour suppressor gene in which mutations lead to an increased risk of breast and ovarian cancers, Mutations in BRCAZ ate alzo linked to an increased risk of pancreatic cancer and an increased isk of prostate cancer in men. Bath ERCAJ and CA2 follow an autosomal dominant inheritance pattern 8 c2e2v “The €282¥ mutation is a common cause of haemochromatoss in the European population, The other ‘mutation reported in European populations as a cause of haemachromatosis is H63D. The condition is autosomal recessive although compound heterozygotes with one C282Y mutation and one H63D mutation can present with symptomatic disease. © Detta-F508 ‘The delta-F508 mutation leads to cystic fibrosis because of a defect in chloride channel function. Ths leads ta abnormally viscous respiratory secretions, pancreatic dysfunction and subfertlity, al of which are hallmarks ofthe disease = P53 P53 a tumour suppressor gene. Mutations in P53 leod to a marked increase inthe incidence of a range of cancers, including ovarian, oesophageal, colorectal head and neck, larynx and lung cancers Previous Question ag Question Feedback nd Review Blog About Pastest Contact Us Help 4 © Pastest 2023 Sy 48 al sot Question 21 of 10 A 41-year-old woman is 28 weeks pregnant with her third child. She is admitted to the Emergency Department with 30 minutes of central crushing chest pain radiating to her left arm. She continues to smoke five cigarettes per day. Her blood pressure is 105/80 mmHg and her pulse is 105 bpm and regular. She is pale and sweating. Her chest is clear and her abdomen is soft and non-tender. Her ankles are not swollen. Investigation Result Normal value Haemoglobin (Hb) 102 gt 115-160 g/t White cell count (WCC) 114 10%" 4-11 x 10% Platelets (PLT) 201 x 10% 150-400 x 10%/1 Sodium (Na°) 143 mmol/l 135-145 mmol/l Potassium (K*) 4.0 mmo 3.5-5.0 mmol/L Creatinine (Cr) 110 umot 50-120 pmol/L Electrocardiogram 3 mm ST elevation in leads Il, IIl and aVF High-sensitivity (hs) troponinT 125 ng/L < 14ngit Which of the following is the most appropriate intervention? A Intravenous (IV) alteplase B_ Oral fondaparinux and ticagrelor C IV glyceryl trinitrate (GTN) D Oral verapamil — Percutaneous coronary intervention, Previous Question Save and Leave End Session Skip Question Blog About Pastest ContactUs Help © Pastest 2023 Question 21 of 100 {A 41-year-old woman is 28 weeks pregnant with her thtd child. She is admitted to the Emergency Department with 30 minutes of central crushing chest pain radiating to her left arm. She continues to smoke five cigarettes per day. Her bload pressure is 105/60 mmbg and her pulse is 105 bpm and regular. She is pale and sweating, Her chests clear and her abdomen is soft and non-tender. Her ankles are not swollen, Investigation Resutt Normal value Platelets (PLT) 201 x 10%” 150-400 x 10° Sodium (Na*} 143 mmol 135-145 mmol Potassium ik") 4.0 mmol 35-50 mmott Creatinine (C7 110 pmol 50-120 pmol Electrocariogram 3 mm ST elevation in leads Il and aE High-sensitivity (hs) ropeninT 125 agi Which ofthe folowing is the most appropriate intervention? A Intravenous (1) alteplase B Oral fondaparinux and ticagrelor © Walycoryt trinitrate (GTN) D Oral verapamil Explanation ° This patient has clear ST elevation inthe inferior leads, a marked elevation in troponin and symptoms consistent with an inferior Ml. Percutaneous coronary intervention is, therefore, the optimal next step, Although coranaty artery dissection is more common in pregnancy, occlusion is potentially more likey here sven the patient's tobacco smoking and the fact that pregnancy itself isa hypercoagulable state, Total coronary artery thrombosis is thought to be responsible fora presentation with Ml in up to 1796 of pregnancy-related cases. ‘A Intravenous (lV) alteplase Thrombolytic therapy is relatively contraindicated in pregnancy. The majority ofits use isin patients with acute stroke and those with massive pulmonary embolism, Although it is thought that the passage of ‘thrombolytic therapy across the placenta is minimal, preterm delivery, etal loss, placental abruption and other pregnancy complications have been reported. Fora likely myocardial infarction (Mi) as here percutaneous coronary intervention is preferred B Oral fondaparinux and ticagrelor In this case, antiplatelet therapy, combined with 2 factor X inhibitor, I nat going to be effective in managing an ST elevation Ml (STEM). Aspirin fs recognized to cause premature closure of the ductus arteriosus and it ‘may also cause premature feta oss, Data onthe risks of agents such as clopidogrel or ticagrelor are not known. although a factor X inhibitor such as fondaparinux does have a role in managing acute Mi itis Unlikely to be sufficient treatment here € Walyceryt trinitrate (GTN) \Vasodilation with nitrates is an option for coronary artery spasm, although here, with ST-clevation, rather than ST depression, significant chest pain, raised troponin and a history of smoking, coronary artery occlusion Is much more tel. Doral verapamil Calcium channel antagonists such as verapamil are relatively safe in pregnancy and can be used chronically for control of atrial tachycardias. In coronary artery vasospasm, they may be of value, butin this case, ST elevation inthe inferior leads is indicative of an Ml Previous Question ag Question Feedback nd Review Blog About Pastest Contact Us Help © Pastest 2023 Answered Flsaged a Oo @ 0 a Oo Qa U 9s 0 6 is) a 0 a8 Op ° ete Carey Sone nd Myce n Question 22 of 10: ‘A 27-year-old man who has type | diabetes presents to the clinic complaining of patchy hair loss affecting his scalp and beard. There is no scarring and the areas of hair loss are bordered by shortened broken-off hairs. Which of the following is the most likely cause of the patient's hair loss? A Alopecia areata B Anagen effluvium C Scleroderma D__ Telogen effluvium — — Trichotillomania Previous Question Save and Leave End Session Skip Question Blog About Pastest ContactUs Help © Pastest 2023 Question 22 of 100 {A27-year-old man who has presents to the clinic complaining of = There is and the areas of hair oss are Which ofthe following is the most likely cause of the patients hairloss? ‘Anagen effiuvium © Scleroderma D.Telogen effiuvium £ —Wiehotilomania Explanation o “The aetiology of alopecia areata is unknown, although itis recognised to occur inpatients with pre-existing ee autoimmune conditions such as type | diabetes. Its associated with patchy, non-scarring hairloss, as seen here. The short hairs around the boundary ofthe area of hair oss are known as exclamation mark hairs. Potent topical steroids may be of value in limiting further hair loss and, in some cases, can lead to recovery, though they cannot be used on the face. B Anagen effiuvium ‘Anagen effluvium occurs when the har growth cycle is arested in the anagen phase. It occurs when cancer chemotherapy, radiotherapy or immunosuppression ead to rapid hai loss. Doxorubicin and cyclophosphamides are two agents that are well known to lead to hair loss, Recovery of hair growth usually begins within afew months of ceasing treatment. C Scleroderma Scleroderma is associated with scarring hair loss, nat with hair oss without scarring as reported here, In addition to the here 3k of evidence of scarring, there are also no other features of systemic sclerosis reported D_Telogen effiuvium Telogen effluvium occurs when physiological or hormonal stress causes many hairs to move into the telagen phase, As new hairs appear in the anagen phase, it causes the telogen hairs to be pushed out, between one ‘and six months after the original stressor, Patients with telogen effluvium often present to their General Practitioner complaining that they are shedding more hair than usual. Itcan be associated with childbirth, anorexia and sudden intentional weight loss, eg after bariatric surgery E Wchotilomania “Tichotomania occurs when patients have a compulsion to pull out theirhair. Although patients with ‘richotllomania can pull out hair from one area on their scalp itis not associated with patchy hair loss from ‘the beard tis associated with other psychiatric disorders, including a formal diagnosis of obsessive compulsive disorder and can lead to intestinal obstruction due to the formation ofa ball of hair once itis swallowed, Previous Question ag Question Feedback nd Review Blog About Pastest Contact Us Help © Pastest 2023 Q a @ 3 Qs 9s ry q 8 ere) Ge 0 7 0 ae Sy 4B vir byl trot Question 23 of 10: 4 48-year-old man who is a known alcoholic with advanced cirrhosis presents to the Emergency Department with increased ascites and abdominal pain. He also has a fever and has been confused over the past 24 hours, according to his wife. His blood pressure is 95/60 mmHg and his pulse is 95 bpm and regular. His temperature is 39 °C. His abdomen is generally tender, with tense ascites. An ascitic tap reveals a neutrophil count of 270 neutrophils/tl. Which of the following is the most likely cause of this patient's worsening ascites? A Escherichia coli B Klebsiella pneumoniae Pseudomonas aeruginosa D Streptococcus pyogenes — Streptococcus viridans Previous Question Save and Leave End Session Skip Question Blog About Pastest ContactUs Help © Pastest 2023 axe SY 4 ao Question 23 of 100 Cor ‘A d@-year-old man wha is a known presents tothe Emerge Department wi He als has fever and has been SST (BEBURAERBUEE, according to his wie His bloodpressure is 95/60 mmHg and his pulse is 95 bpm and regula, His temperature is 39°C. His abdomen is generally tender, with tense ascites, RASERIERSTOVSES Which ofthe following is the most likely cause of this patients worsening ascites? . cy Klebsiella pneumoni Answered Flagged © Peeudemonas aeruginosa a Oo @ D Streptococcus pyogenes Lal @ Oo E Streptococcus viridans a Cr Oo Explanation ° es Oo a - qr 0 Gram-negative bail cause around three-quarters of cases of spontaneous bacterial peritonitis and E.coli ‘makes up around 50% of those cases in pationts with end-stage cirrhosis. It fit withthe symptoms here of. 8 06 pyrexia, worsening ascites and abdominal pain and a neutrophil count of above 250/uL. Cefotaxime 2.9 intravenously three times per day isthe intervention of choice, Asi lui i usually cuttured in blood culture bottles to improve the yield of potential organisms. Mortalty from spontaneous bacterial peritonitis high at above 40% and the prognosis is particularty poor in patients with concomitant renal impairment. a B Klebsiella pneumoniae a Klebsiella infection is the next most common cause of Gram-negative spontaneous bacterial peritonitis after Se to infection with E coli lt appears that patients with chronic alcoholism are particulary suscept Klebsiella, Peeudomonas aeruginosa Pseudomonal infections are responsible for 10%6 or less of cases of spontaneous bacterial peritonitis. Where Pseudomonas is suspected or cultured, cefotaxime, as monotherapy isnot considered as adequate antibiotic coverage and aminoglycoside, would usually be added, D Streptococcus pyogenes “Streptococcus pneumoniae, not S. pyogenes, I a common cause of spontaneous bacterial peritonitis, making Up around 1296 of cases. Cefotaxime is adequate cover against streptococcal infection E Streptococcus viridans S vinidansis a ss common cause of spontaneous bacterial peritonitis than S. pneumoniae, making up around 9% of cases. Like S. pneumoniae, iis sensitive ta treatment with cefotaxime. Previous Question ag Question Feedback nd Review Blog About Pastest Contact Us Help q © Pastest 2023 ae Sy B vtr yl ry sot Question 24 of 10 ‘A 32-year-old woman with multiple sclerosis (MS) presents to the clinic for review. She has cerebellar ataxia, weakness affecting her left leg and some slurring of her speech which is long-standing. Her husband is concerned because she appears apathetic, is sleeping all the time and not eating. She looks depressed. Her body mass index is 19.5 kg/m?. Her General Practitioner referred her for cognitive therapy, although this does not appear to have helped her symptoms.A short course of citalopram resulted in gastrointestinal side effects and she is not keen to retry this agent. Investigations: Investigation Result Normal value Haemoglobin (Hb) 132 gf 115-160 gil White cell count (WCC) 71x 10% 4-11 x 10% Platelets (PLT) 381 x 10% 150-400 x 10° Sodium (Na*) 142 mmol/L 135-145 mmol/l Potassium (K*) 3.6 mmol/l 3.5-5.0 mmol/L Creatinine (Cr) 117 pmo 50-120 ymol/t Which of the following is the most appropriate medication for this patient? A Bupropion B _ Escitalopram C Imipramine D Modafinil E Venlafaxine Previous Question Save and Leave End Session Skip Question Blog About Pastest ContactUs Help © Pastest 2023 Question 24 of 100 Co ‘A.32-year-old woman with FOEBIERERBRIETMAS) presents tothe clinic for review. She has cerebellar ataxia, weakness affecting her left leg and some slurring of her speech whichis long-standing, Her husband is concemed because she appears sie ISS aU UM AC OTESUNG. She Looks depressed Her body mass index is 19.5 kglm®. Her General Practitioner referred her for cognitive therapy, although ti A short course of citalopram resulted in gastrointestinal side ‘ effects and shes not keen to retry this agent. 5 Investigations . Investigation Result Normal value ° Haemoglobin (Hb) 1329 115-160 91 ‘White cell count (WCC) 7.110% 4-11 x 10% bed Platelets PLT) 381x104 150-400 1 ae aaa Sodium (Ne*) 142 mmovt 135-145 molt = ry Potassium i) 3.6 mmovt 35-50 mmott @ Oo Creatinine C7 117 pout 50-120 pmol a Oo Which of the following i the most appropriate medication for this patient? 8 7 4 5 Oo A Bupropion Go 0 a Oo B Esctalopram 98 Oo ° Imipramine D_Modafinit — 5 tie roi eS n Explanation * ‘This patient has depression and significant somnolence. This puts her in a group likely to respond well to ‘treatment with venlafaxine, whichis a serotonin and noradeenaline reuptake inhibitor which interferes with ‘the reuptake of serotonin and noradrenaline, Although the National insttute for Health and Care Excellence (NICE) quidetines for chronic conditions and depression recommend a selective serotonin reuptake inhibitor, siven the significant somnolence here, venlafexine may be more effective ‘A Bupropion Bupropion is not usually pre te ribed forthe treatment of depression in the United Kingdom but is used as a ment for smoking cessation. It may be of value if used as an anti-depressant for patients with MS when there are symptoms of sexual dysfunction as well as depression. 8 _Escitalopram Escitalopram isa preferred option for treating depression in patients with MS, particularly where there is snificant anxiety, thas a rapid onset of action and requires limited titration, which makes it deal for Patients who may be taking multiple medications. C Imipramine Imipramine isthe usual first-line option for patients with urinary incontinence, particularly when there is rocturia. In this patient’ situation, it may increase somnolence D_ Modafinit “Modafinil is a dopamine reuptake inhibitor primarily used in the treatment of narcolepsy and attention deft hyperactivity disorder Previous Question ag Question Feedback nd Review Blog About Pastest Contact Us Help 4 © Pastest 2023 ae Sy Avi byl teat Question 25 of 10 A 54-year-old farmer presents to his General Practitioner for review. He has chronic joint pain, abdominal distension, flatulence and diarrhoea, accompanied by gradual weight loss over the past few months, He has also had a chronic cough and intermittent fevers. His blood pressure is 110/70 mmHg and his pulse is 85 bpm and regular. He has mild abdominal distension, although his body mass index is low at 21 kg/m?, Investigations: Investigation Result Normal value Haemoglobin (Hb) 102 git 135-175 gil White celt count (WCC) 81x 107 4-11 10" Platelets (PLT) 231 x 10% 150-400 x 10%/t Sodium (Na*) 142 mmol/L 135-145 mmol/L Potassium (K*) 3.9 mmol/L 3.5-5.0 mmol/L Creatinine (Cr) 110 pmoV/t 50-120 pmo C-reactive protein (CRP) 49 mg/l <10 mg/l A small bowel biopsy was performed and results were: villous atrophy, infiltration of lamina propria with periodic acid-Schiff (PAS)- positive macrophages. Which of the following is the most likely diagnosis? A Behcet's disease B Coeliac disease C Familial Mediterranean fever (FMF) D_ Small bowel tymphoma E Whipple's disease Previous Question Save and Leave End Session Skip Question Blog About Pastest ContactUs Help © Pastest 2023 Question 25 of 100 ‘A 54-year-old farmer presents to his General Practitioner for review. He has SFSRIE SIE Ia) SSSR lover the past few months. He also had 3 His blood pressure is 110/70 mmHg and his pulse is 85 bpm and regular. He has mild abdominal distension, although his body mass index is tow at 21 kai? Investigations: Investigation Result Normal value ‘Haemoglobin (Hb) za 135-475ait White cell count (WCC) 81x10" 411 10% Platelets (PLT) 23110 150-400 10% Sodium (Na") 142 mmout 135-145 mmout Potassium (K*) 3.9. mmol 35-50 mmol Creatinine (Cr 110 umoit 50-120 pmol Which ofthe following is the most tkely diagnosis? A Behcet's disease 8 Coelic disease C__Familat Meeiterranean fever (FMF) Small bowel lymphoma Je |rnmrsee Explanation o ‘Thi patient's symptoms, including weightloss, joint pains, chronic cough and an iritable bowel syndrome: type picture, fit well with the diagnosis of Whipple's disease. The presence of partial villous atrophy and PAS-positive macrophage infitration further supports the diagnosis. Antibiotic recommendations vary for Whipple's disease; one typi teimethoprim-sulfamethoxazole regimen is tivo weeks of ceftriaxone followed by up ta one year of A Bencet’s disease Behcet's cisease is associated with oral and genital ulcerations and anterior uveitis; itis more common in patients of Mediterranean or Middle Eastern origin It's associated with abdominal pain, distension and diarmnoea, although not with villous atrophy or PAS-positive macrophages. B Coeliac disease Although cosliae disease does resultin villous atrophy. it does not Lead to infiltration ofthe lamina propria with PAS-positive macrophages. Other features of coeliac disease such as dermatitis herpetiformis are also absent here C Familial Mediterranean fever (FMF) Familial Meciterranean fever (FMF) is a hereditary inflammatory disorder that is associated with episodes of abdominal pain, which ate, on accasion, so severe that they prec even exploratory laparotomy. Itis also associated with joint pains and episodes of pleurtc chest pain. but is nota cause of PAS-positive macrophage infiltration inthe small bowel tate admission under surgical teams and Small bowel lymphoma ‘Small bowel lymphoma can occur in patients with long-term coeliac disease, particularly when their cise is poorly controlled tis associated with lymphooytc, rather than PAS-positive macrophage, infiltration, Previous Question ag Question Feedback nd Review Blog About Pastest Contact Us Help © Pastest 2023 ae Q Answered Flagaed a 0 a C 3 0 Qs C 95 D 6 0 qr 0 98 D6 ° ae Sy Avi byl teat Question 26 of 10 A 49-year-old man presents to the Dermatology Clinic with increased skin pigmentation over the back of his neck and axillae, as well as affecting both groins. He had also begun to develop pigmentation over the soles of his feet. He tells you he has lost weight over the past three months and is generally off his food. You note skin thickening and dark pigmentation over the areas concerned. His body mass index is 23 kg/m?. Investigations: Investigation Result Normal value Haemoglobin (Hb) 102 g/t 135-175 gf White cell count (WCC) 89 x 10% 4-11 x 10% Platelets (PLT) 303 x 10% 150-400 x 10%/1 Sodium (Na*) 142 mmol 135-145 mmol/l Potassium (K*) 43 mmol 3.5-5.0 mmol/l Creatinine (Cr) 110 pmol/L 50-120 pmol/L Erythrocyte sedimentation rate (ESR) 75 mm/hour < 10 mm/hour Which of the following is the most likely diagnosis? A Gastric adenocarcinoma B Gastric MALToma C Prostate cancer D Small cell tung cancer E Squamous cell lung cancer Previous Question Save and Leave End Session Skip Question Blog About Pastest ContactUs Help © Pastest 2023 Question 26 of 100 Cog ‘A.49-year-old man presents to the Dermatology Clinic with He had also begun to rt 1 tolls you ne has lost weight over the pact three months and is generally off his food. You nate Ra skin thickening and dark pigmentation ever the areas concerned, His body mas index is 23 kan? Investigations: | Investigation rest Rermclaea Sedum (No 2A2mmolA 136-145 mmott Answered lagged a 0 75immmou «10 mmour ae : @ D Wich ofthe folowing is the most they gnosis? a t 5 Oo Js [corcowcnoms | “ - @ 8B Gastric MALToma 5 8 vo ° C Prostate cancer 1D Small cell tung cancer ev a E Squamous cell tung cancer Explanation 2 G a ‘This patient has gastric adenocarcinoma, as exemplified by weight loss, anaemia and progressive development of acanthosis nigricans, which is recognised to occur in patients with gastric cancer recognised to occur in tumours ofthe gastraintestnal tract, lung, uterus, ovaries and urinary tract. Excision of the undertying tumour can lead to the resolution af the skin lesions. Iris atco B Gastric MALToma Gastric MALTomas are seen in association with chronic Helicobacter infection. They often resolve when Helicobacter infection is cleared. They are not associated with the development of acanthosis nigricans. C Prostate cancer “Malignant acanthosis niricans is very rarely seen in patients with prostate cancer. It is seen much more commonly in patients with gastric adenocarcinoma, Small cell tung cancer ‘Small cell lung cancers associated with the development of synrome of inappropriate antidiuretic hormone. secretion and can also lead to the development of Cushing syndrome. A range of skin changes can be seen in Patients with small elt lung cancer. including psoriasis-type and acneiform rashes, but not usually acanthosis nigricans. E Squamous cell lung cancer ‘Squamous cal carcinoma ofthe ung is associated withthe production of parathyroid hormone-related protein which leads to the development of hypercalcaemia. It does not usualy resultin the development of acanthosis nigricans. A pemphigoid-type rash has been described in some patients presenting with squamous cell carcinoma of the bronchus. Previous Question ag Question Feedback nd Review Blog About Pastest Contact Us Help 4 © Pastest 2023 Question 27 of 10 ‘A 39-year-old off-duty fireman is admitted to the Emergency Department after having rescued a small child from a house fire. He is thought to have inhaled a mix of carbon dioxide (CO2) and carbon monoxide (CO). What do peripheral chemoreceptors do in CO poisoning? A Drive a decrease in ventilatory rate B Drive an increase in ventilatory rate C Drive an increase in depth of inspiration D Drive a decrease in depth of inspiration E Have no effect on ventilation Previous Question Save and Leave End Session Skip Question Blog About Pastest ContactUs Help © Pastest 2023 axe SY AB tr J Question 27 of 100 eee ‘A 39-year-old off-duty fireman is admitted to the Emergency Department after having rescued a i ia 3 Oo Explanation 2 Carbon monoxide levels do not impact the activity of peripheral chemoreceptors. In this situation, where 6 0 inhalation and central chemoreceptor activity. One possible driver for respiratory depression after a house Q7 0 ° ‘on the ventilatory rate. Ev B Drive an increase in ventilatory rate a An increase in ventilatory rate is driven by hypoxia, ut inthe case of CO retention, peripheral chemoreceptors fal to detect resultant tissue hypoxia, Patients presenting with CO poisoning do, however, usually present with dyspnoea. This may well be due to CO, retention, the response to which isnot blunted by accumulation of CO, € Drive an inreasein depth of inspiration Carbon dioxide drives an increased depth of inspiration via stimulation of central chemoreceptors in the pons. and medulla, This s because ofa change in dissolved CO; levels in the cerebrospinal fluid. CO levels do not have the same effect, D_ Drive a decrease in depth of inspiration Decreased depth of inspiration is driven by respiratory depressants such as benzodiazepines. CO excess does not have this effect Previous Question ag Question Feedback end Review Blog About Pastest Contact Us Help 4 © Pastest 2023 Question 28 of 10: You conduct an early-phase study of a new agent for the treatment of hypertension. You measure the blood pressure of 30 subjects before and after a test dose of the new drug. Which of the following is the most appropriate statistical test to measure the effect of the drug? A Analysis of covariance (ANCOVA) test B Kruskal-Wallis test C Mann-Whitney U test D Paired ttest E Spearman's rank test Previous Question Save and Leave End Session Skip Question Blog About Pastest ContactUs Help © Pastest 2023 Question 28 of 100 ‘You conduct an early-phase study of = SSSR NE=SUREALSNVSERIEREIN You measure the blood pressure of 30 subjects before and after atest dase f the new drug Which of the following is the most appropriate statistical test to measure the effect ofthe drug? Analysis of covariance (ANCOVA) test 8 Kruskal-Wallis test Mann-Whitney U test BS 5 ‘Spearman's rank test Explanation 2 ere “The paired testis used for normally distributed data (ike blood pressure) and compares two groups, in this case, before and after treatment with the new antihypertensive It can be used to confirm or reject the null hypothesis that blood pressure isthe same or significantly diferent) before and after treatment, A Analysis of covariance (ANCOVA) test ANCOVA is not appropriate here. ANCOVA evaluates whether the mean of a dependent variable isthe ‘across levels ofa categorical independent variable, which may be a medical intervention ora treatment. ft does this while controling for the effects of other continuous variables that are not f interest: tis not appropriate here given that we are measuring blood pressure, which is normally distributed at two time points in the same patient hence, a pared t-test is more appropriate B Kruskal-Wallis test This is a non-parametric test to determine if two samples originate from the same dlstribution. In this case, blood pressure is normally distributed and we are testing the effect ofan intervention inthe same patients hence, a pared t-test is more appropriate Mann-Whitney U test ‘The Mann-Whitney U tet is another non-parametric statistical test used to test the null hypothesis that, for randomly selected values from two populations, the probability ofA being greater than Bis the same as the probability of 8 belng greater than A. E Spearman's rank test “The Spearman's rank testis used for non-parametric data to measure the correlation between two variables, {An example of two sets of data where ths test would be useful would be the examination ofthe relationship between hours of computer gaming as a child and achieved intelligence quotient (IQ). tis not appropriate for use in looking at change in blood pressure, a normally distributed variable, before and after an intervention, Previous Question ag Question Feedback nd Review Blog About Pastest Contact Us Help © Pastest 2023 a ty asy Q Answered Flagged qa oO Q oO 3 Oo Qs oO 95 0 6 0 q Oo 8 Oe Question 29 of 10 74-year-old woman presents to the hospital for a routine appointment for hypertension management. Her blood pressure is 135/82 mmHg and her pulse is 72 bpm and regular. She has no urinary symptoms. Urine dipstick is positive for protein and nitrites. Which of the following is the most appropriate next step? A Reassure B Repeat sample in one week C Send sample for microscopy, culture and sensitivity D Treat with nitrofurantoin E Treat with trimethoprim Previous Question Save and Leave End Session Skip Question Blog About Pastest ContactUs Help © Pastest 2023 Question 29 of 100 eee ‘A 74-year-old woman presents tothe hospital for a for hypertension management. Her a blood pressure i 135/82 mmHg and her pulse is 72 bpm and r Which ofthe following is the most appropriate next step? pete . 8 Repeat sample in one week Send sample for microscopy, culture and sensitivity Answered Flagged 2 D_ Treat with ntrofurantoin 9 Oo a Oo E Test with trimethoprim @ Oo Explanation 2 Co ( a « . 96 Oo ‘Asymptomatic bacteriuria is common inthe elderty and isnot associated with an increased risk of symptomatic urinary tract infection. Treatment isnot necessary and the patient should be reassured, a Oo Management of asymptomatic bacteriuria in pregnant women, however, is always treated with antibiotics. 98 ( ° Repeat sample in one week In the absence of symptoms ofa urinary tract infection, there is no value in repeating the sample. Many elderly men and women have episodes of asymptomatic bacteriuria and it caries ne prognostic significance. —_—v- C Send sample for microscopy, culture and sensitivity a If this patient had symptoms, sending the sample for microscopy, culture and sensitivity would be appropriate, As the patient is asymptomatic, sending the sample for culture isnot required D__Trest with nitrofurantein Empirical treatment with itrofurantoin could be considered in a patient with dipstck-positive bacteriuria and symptoms In tis case, with no symptoms, antibaties are not required. Instituting antibiotic therapy could increase the chance of resistance occurring in the future. E Treat with tmethopxim Trimethoprim is a fisttine option for the treatment of urinary tract infection in the elderly. n this patient i should not be used because there are no symptoms of urinary tract infection. Use of antibiotics in the elderly without symptoms of urinary tract infection has no positive impact on outcomes and may actually lead to an increased risk of resistant sepsis Previous Question ag Question Feedback end Review Blog About Pastest Contact Us Help © Pastest 2023 ae Sy BB vty Jay Yr ae Question 30 of 10: A 62-year-old man is reviewed in the Emergency Department following a syncopal episode. The ambulance crew noted him to be in ventricular tachycardia, which reverted spontaneously. He suffered a delayed presentation of an anterior ST-elevation myocardial infarction (STEMI) some eight weeks earlier. He currently has no chest pain. His blood pressure is 122/82 mmHg and pulse is 88 bpm and regular. There are no signs of cardiac failure. Investigation Result Normal value Haemoglobin (Hb) 145 g/t 135-175 git White cel count (WCC) 7x 10% 4-11 x 10% Platelets (PLT) 182 x 10/1 150-400 x 102 Sodium (Na*) 141 molt 135-145 mmol/L Potassium (K*) 4.9 mmol/L 3.5-5.0 mmol Creatinine (Cr) 110 umott 50-120 pmol/l High-sensitivity (hs) troponin T 12 ngft <14ngh An electrocardiogram (ECG) was performed and found anterior Q waves, persistent elevation in V1 to V3, anterior T wave inversion. Which of the following is the most likely cause of the patient's ventricular tachycardia? A Coronary artery vasospasm B___ Ischaemic cardiomyopathy C Left ventricular aneurysm D Long QT syndrome — New STEMI Previous Question Save and Leave End Session Skip Question Blog About Pastest ContactUs Help © Pastest 2023 Question 30 of 100 Co {A62-year-old man is reviewed in the Emergency Department following 2 SRESBBIRBEBEE The ambulance crew noted him tobe in ventricular tachycardia, which reverted spontaneously. He sufered « aa some eight weeks earlier, He currently has His blood pressure is 122/82 mmHg and pulse fs 88 bpm and regular. There are no signs: of cardiac failure ‘ Investigation Result Normal value : Haerogloin (Ho) M501 135-1750 a ‘White eat count (WCC) 74 %10% #11«10 . Platolets (PLT) 182.102" 150-400 x 10°” : 5 ob bb bb Sodium (Na) 141 mmott 135-145 mmolA Potassium ik") 49 mmott 35-50mmolt ees Creatinine (C7 110 pmo 50-120 molt i as High-sensitivity hs) 12g < 14 ngh Q . {An electrocardiogram (ECG) was performed and found anterior Q waves, persistent elevation in V1 to V3, @ a anterior T wave inversion. a mw Which ofthe following is the most likely cause ofthe patient's ventricular tachycardi Os Oo : 6 . A Coronary artery vasospasm q Oo Ischaemic eariomyopathy es Lio} D Long QT syndrome E _NewSTEM! Explanation * ‘A new presentation with ventricular tachycardia against a background of a recent anterior myocardial infarction (Ml) and limited ST elevation in leads V1 to V3 is suggestive ofa left ventricular aneurysm, especially given there is echocardiography and magnetic resonance imaging, which can further elucidate cardiac anatomy. Aneurysm excision is considered in patients with recurent ventricular arrhythmia est pain and no elevation in troponin. Diagnosis is usually made by A Coronary artery vasospasm Coronary artery vasospasm is usually associated with ST depression rather than the ST elevation seen here, In the absence of arse in traponin or chest pain, the ST elevation seen here is more likely to be associated with a left ventricular aneurysm. Coronary artery vasospasm is commonly managed with nitrates when patients present acutely with chest pain and ST depression. B Ischaemic cardiomyopathy Ischaemic cardiomyopathy isa cause of ventricular tachycardia post-Mi, although the features here are more suggestive ofa left ventricular aneurysm, given the localised ST elevation, A number of ECG abnormalities are recognised in association with ischaemic cardiomyopathy, including poor R wave progression and left bundle branch block Leng QT syndrome This patients not taking any QT-prolonging drugs and has specific ECG features suggestive ofa left ventriear aneurysm. QT prolongation is associated with torsades de pointes ventricular tachycardia and is ‘managed in the fist instance with intravenous magnesium loading. & NewSTEM! “There is no chest pain to suggest a new episode of myocardial ischaemia and the troponin is well within the normal ange. The ECG changes also raise the possibilty ofa left ventricular aneurysm. A new STEMI is, ‘therefore, much less likely than an aneurysm. ag Question Feedback nd Review Blog About Pastest Contact Us Help © Pastest 2023 ae Sy B vit bol ry sod Question 31 of 10 A 33-year-old florist presents to the Emergency Department with a red, shiny, indurated patch on her face just to the left-hand side of her nose, which she says began as some skin erythema two days earlier. She tells you that she had an episode of sinusitis the previous week which seemed to be resolving. The skin over the area of reddening is oedematous, hot and painful to touch. Investigation Result Normal value Haemoglobin (Hb) 92 g/t 115-160 g/t White cell count (WCC) 117 x 10% 4-11 x 10% Platelets (PLT) 76 x 10% 150-400 x 10°/t Sodium (Na*) 144 mmoUt 135-145 mmol/L Potassium (K*) 5.4 mmol/l 3.5-5.0 mmol/L Creatinine (Cr) 215 pmol/l 50-120 umol/t Which of the following is the most appropriate intervention for this patient? A Oral clarithromycin B Oral co-amoxiclav C Oral fluctoxacillin D Oral vancomycin E Topical Fucidin® ointment Previous Question Save and Leave End Session Skip Question Blog About Pastest ContactUs Help © Pastest 2023 Question 31 of 100 Investigation Result Normal value Haemoglobin Hb) 92h 115-160 gf White cell count (WCC) 117 «10% 4-11 10° Platelets (PLT) 76% 10% 150-400» 10°” Sodium (Na*) 144 mmovt 135-145 mmol Potassium (K") 5.4 mmol 35-50 mmol Creatinine (CA 215 pmout 50-120 pout Which ofthe following i the most appropriate intervention for this patient? A Oral clarithromycin C Oral fuctoxacitin Doral vancomycin E Topical Fucidin® ointment Explanation o Je fonwemen Co-amoxiclav is the intervention of choice for erysinelas which is near to the nase or eye, Although the most Uely causative organism is group A Streptococcus, the wider spectrum of covera including against anaerobes, is desired because of the postion ofthe lesion, A higher dose of 625 mg three ‘times daily is usually prescribed. Where the infection is rapidly spreading, or there are marked systemic symptoms, intravenous therapy ie instigated, from co-amoxielay, A Oral clarithromycin Cra clarithromycin is usually considered for erysipelas when patients are allergic to penicillin and is not used as monotherapy where lesions exist near to the nose or eye. In this situation, broader coverage is required and clarithromycin is usualy deployed in combination with metronidazole, C Oral fuctoxaciin Cra fuctoxacillin ata high dose is recommended by guidelines only when the erysipelas lesion is notin close proximity to the eye or nose. Ata dose of 2 once daily, its adequate to cover against both staphylococcal and streptococcal infections. Some practice guidelines suggest it can be given in conjunction with peniciin V, although there is no evidence to support this practice, D_Oralvancomycin ‘Vancomycin is considered for patients thought to be at risk of methicilin-resistant Staphylococcus aureus (MRSA) infection, although delivered orally, vancomycin has no systemic bioavailability. Where MRSA is suspected, both vancomycin intravenous and linezolid are potential options, a is teicoplanin, E Topical Fucdin® ointment “Topical Fucidin® ointment is most effective in the treatment of staphylococcal skin infection and in clearing MRSA in patients and healthcare staff with chronic carriage. In this case, with well-established infection, systemic therapy with co-amoxiclav is a more appropriate intervention, Previous Question ag Question Feedback nd Review Blog About Pastest Contact Us Help © Pastest 2023 a a @ Ce) Qs 95 q 8 Answered a GB ioe Flagged ae Sy Bitte! tr aot Question 32 of 10 A 23-year-old woman presents to the clinic complaining of pain on passing urine and joint pains affecting her right knee and left ankle. She also has lower back pain and obtained chloramphenicol ointment from the pharmacy two days earlier for conjunctivitis. She also tells you she had unprotected sexual intercourse with a man she met on a dating app some two weeks earlier. Her temperature is 37.9 °C, blood pressure 115/82 mmHg and pulse 70 bpm and regular. You confirm evidence of oligoarthritis. Investigations: Investigation Result Normal value Haemoglobin (Hb) 132 g/t 115-155 g/l White cell count (WCC) 7.2x 10% 4-11 x 10% Platelets (PLT) 301 x 10% 150-400 x 1097/1 Sodium (Na*) 140 mmol/L 135-145 mmol/L Potassium (K*) 4.2 mmol/L 3.5-5.0 mmol/L Creatinine (Cr) 95 umollt 50-120 umol/t C-reactive protein (CRP) 55 mg/l < 10 mgit Which of the following is the most likely cause of her arthritis? A Fibromyalgia B — Gonococcal arthritis Cc Gout D Reactive arthritis, — Rheumatoid arthritis Previous Question Save and Leave End Session Skip Question Blog About Pastest ContactUs Help © Pastest 2023 suey OB et ex! todd Question 32 of 100 Cog ‘A 23-year-old woman presents tothe clinic complaining of pain on passing urine and joint pains aff " army two dys er for ounce Se lo tl you se ed nprtactd exuarcours tha eee a eres catareee ere ie eee eae a eee es stele netic at era aoe aT aaa eee a i lovestigatons : Investiation Fest Ratan ‘ Whe cal count (WC) 72x10" eapei0%n Protas eu) 301 «10% 150-400 10% Seu Na 140 mmett 135-145 melt ee Feta 42mmott 35-50 met a 0 Creatine (x 25 pelt 50-120 mai a 5 Corwaetive protein (CRP) sing <10mph 3 oO Which ofthe fllowing isthe most ikely cause of her arthritis? ry oO | Your enower wesiocomect @ 5 oe 0 5 ees @ D B_ Gonocccl ars QS vo ° © | con ° 0 eametoi atts q Explanation * © 6 4 Jimercoe | : : Reactive arthritis is seen around two weeks ater infection with Chlamydia recognised ast cause of the disease. The timing ofthe presentation with arthritis ite withthe episode of unprotected sexual intercoure, as do the other features seen here including dysuria and conjunctivitis Inthe event that Chlamydia is identified as the cause of the patient's presentation, it should be treated with doxycycline 100, mg twice dally for seven days, although itis nat thought that antibiotics shorten the course of arthritis. Non: steraidals are the intial ant-inflammatory intervention of choice to manage joint pains. A Fibromyalgia Fibromyalgia is associated with chronic widespread pain with multiple individual painful points where the symptoms of pain are worsened by local pressure, Its also assaclated with symatoms of tiredness and lethargy t does not ft withthe clinical picture seen here 1B Gonococcal arthritis Gonocaccat infection can result in oigoartirts although more marked joint inflammationfefusion would be expected. In generalised gonococcal infection a bigger risen CRP and neutrophilia would usually be seen Gout Gout usually presents with mengarthrits and occurs in multiple episodes. It is alzo unusual for a patient aged as young as 23 years to present with gout and other features, such as urethritis and conjunctivitis, do rot occur in conjunction with an episode of urate arthropathy. Rheumatoid arthritis Rheumatoid arthritis is more likely to present with multiple smal, inftamed joints rather than the oligoarthrtis een here. it may present with uveitis, although symptoms of dysuria would be unexpected Previous Question ag Question Feedback nd Review Blog About Pastest Contact Us Help 4 © Pastest 2023 See ey OP it bal trad Question 33 of 10: 72-year-old man with end-stage oesophageal carcinoma is reviewed on the Medical Admissions Unit by the Palliative Care team for management of his pain control. His current medication is MST Continus 80 mg twice daily, although currently, he is unable to swallow because of his tumour. His blood pressure is 112/72 mmHg and his pulse is 80 bpm and regular. He is thin, with a body mass index of 20 kg/m? Investigations: Investigation Result Normal value Haemoglobin (Hb) 102 g/t 135-175 g/t White cell count (WCC) 9.1x 10% 4-11 x 10% Platelets (PLT) 391 x 10% 150-400 x 10% Sodium (Na*) 143 mmol/L 135-145 mmol/l Potassium (K*) 5.1 mmol 3.5-5.0 mmol Creatinine (Cr) 132 pmol 50-120 ymol/t Which of the following is the most appropriate dose per 24 hours for a subcutaneous morphine pump? A 80mg B 40mg Cc 20mg D 10mg — 5mg ee Previous Question Save and Leave End Session Skip Question Blog About Pastest ContactUs Help © Pastest 2023 Question 33 of 100 {A 72.year-old man with end-stage oesophageal carcinoma is reviewed on the Medical Admissions Unit by ‘the Palliative Care team for management of his pain control Mis current medication is MST Continus 80 mg twice daily although currently, he is unable to swallow because of his tumour, His blood pressure is 11272 ‘mmHg and his pulse is 80 bpm and regular, He is thin, with a body mass index of 20 kon Investigations: Investigation Result Normal value White cll count (WCC) 91x 10% 41110 Platelets (PLT) 391 x10" 150-400 x 10° Sodium (Na") 143 mmod 125-145 mmol Which ofthe following is the most appropriate dose per 24 hours for a subcutaneous morphine pump? c 20mg D 10m@ — 5mo Explanation o Pater “The dose reduction factor for oral to subcutaneous morphine is 50% MST Continus 80 mg twice dallyis a total daily dose of 160 mg oral morphine. This translates to a dose of 80 malay for eubcutaneous morphi B 40mg {A dose of 40 mg represents a fourfold reduction vs the oral morphine dose, This is appropriate for conversion from oral morphine to subcutaneous oxycodone. 20mg ‘A dose of 20 ma represents an eightfold dose reduction vs the oral morphine dose. This i approaching the ‘tenfold dose reduction required to convert an oral morphine dose to subcutaneous hydromorphone. D 10mg {A dose of 10 mg is a 16-fold reduction in dose vs the oral dose of morphine. This is too large a reduction, representing eight times too litte morphine delivered over a 24-hour period = 5mo ‘A dose of 5 mg is a 32-fold reduction in dose vs the oral dose of morphine. This approximate: conversion of oral morphine t subcutaneous alfentanil, which isa reduction of 30 mg the ag Question Feedback nd Review Blog About Pastest Contact Us Help © Pastest 2023 2 Answeres a o a 0 a Oo a 0 @s 0 96 0 a o a8 0 esophagen tunours a

You might also like